• Shuffle
    Toggle On
    Toggle Off
  • Alphabetize
    Toggle On
    Toggle Off
  • Front First
    Toggle On
    Toggle Off
  • Both Sides
    Toggle On
    Toggle Off
  • Read
    Toggle On
    Toggle Off
Reading...
Front

Card Range To Study

through

image

Play button

image

Play button

image

Progress

1/173

Click to flip

Use LEFT and RIGHT arrow keys to navigate between flashcards;

Use UP and DOWN arrow keys to flip the card;

H to show hint;

A reads text to speech;

173 Cards in this Set

  • Front
  • Back
• Patient with hypocardia, etc after tunneling a mediport- cause?
• a) air embolism
• b) hemorrhage
• Air embolism
In carotid angiography, what technique is used?
• 60-80 kVp
• 60-80 kVp
Maximum differential absorption occurs at just above the k-edge
Iodine 33keV; want the majority of the incident spectrum to be above this
Which of the following factors will have the least affect on patient radiation dose in the angiography suite?
• patient-tube distance
• patient-receptor distance
• kVp
B. Pt-receptor distance
Pretransplant evaluation for liver showed filling of hepatic artery from gastroduodenal
• Normal variant
• FMD
• Compression from median arcuate ligament/ cruciate ligament syndrome
Compression from median arcuate ligament
Which of the following statements is true about cardiac vessels? ?
• Vasovasorum gives supply to outer wall
• Intima 5-7 cell layers thick
• Adventitia smooth muscles
• Dissection between media and adventitia
• Smooth muscle in adventitia
a. Vasovasorum supplies outer wall
Gastric varices from splenic vein thrombosis
• Not indication for TIPS[[[[[[[REPEAT QUESTION]]]]]]]]]
What is an established indication for TIPS?
• a) failed esophageal sclerotherapy of bleeding varices
A Brescia-Cimino fistula anastomosis which vessels?
• a) Brachial artery to cephalic vein
• b) Brachial artery to basilic vein
• c) radial artery to basilic vein
• d) radial artery to cephalic vein
natural AV fistula between the radial artery and cephalic vein.
Arteriovenous fistula…what causes failure
• Intimal hyperplasia
• Thrombosis
• Venous anastimosis
• Venous anastomis
Adenosine contraindicated in
• Exercise induced angina
• Previous CABG
• Heart block
• Claudication
c. Heart block
Aortic penetrating ulcer (2 questions)
• Usually in lower thoracic aorta
• Associated with enlargement of aorta
• Type B aortic dissection
• Intramural contrast collection
• Enlarged diameter of aorta
• T: usually in the middle to distal third of the descending aorta
• T: The ulcer is often associated with thickening of the aortic wall… or focal out pouching
• ?: Described as distinct from dissection [though difficult sometimes].
• T: Focal contrast collection projecting beyond the aortic lumen on CT
Progression: penetrating ulcer -> hematoma within the media -> pseudoaneurysm -> possible rupture
Penetrating Atherosclerotic Ulcer of the Aorta: Imaging Features and Disease Concept1 Radiographics. 2000;20:995-1005
Lateral medullary syndrome…which vessel?
• Contralateral PICA
• Ipsalateral PICA
• Superior cerebellar
B. Ipsilateral PICA
Wallenberg’s syndrome-
Loss of pain & temp sense of contralateral body
“ “ “ of ipsilateral face
Ipsilateral Horner’s
Ipsilateral cerebellar deficits
Hoarseness, difficulty swallowing
What is true regarding the ankle-brachial index?
A) numerator contains arm readings
b) denominator contains ankle readings
c) in patients with peripheral vascular disease, ABI decreases after exercise
d) with patients who have peripheral vascular disease, increases after exercise
e) in patients without peripheral vascular disease, increases after exercise
Ans: C. A lower ABI is worse. You want the ABI to be > 0.95 (ie, blood flow to your arms is equal to blood flow to your piggy-wiggys). The ankle-brachial index is an effective screening tool. The tools required to obtain an ankle-brachial index include a blood pressure cuff and a continuous wave Doppler. Blood pressure is measured in both upper extremities, and the highest systolic reading--the first return of Doppler sound as the cuff is deflated--is recorded. The ankle systolic pressure is similarly measured using the dorsalis pedis or posterior tibial arteries. The ankle-brachial index is calculated by dividing the ankle pressure (the higher of the posterior tibial artery pressures) by the brachial systolic pressure (the higher of the two arm pressures). An ankle-brachial index below 0.95 at rest or following exercise is considered abnormal. An ankle-brachial index between 0.8 and 0.5 is consistent with intermittent claudication, and an index of less than 0.5 indicates severe disease.11 In patients with an abnormal ankle-brachial index, testing with segmental arterial pressures and a pulse volume recording before and after exercising to the point of absolute claudication are indicated. http://www.aafp.org/afp/20000215/1027.html.
Embolization, which is true:
• a) need to permanently embolize varicoceles b/c associated with infertility
Embolization, which is true:
• a) need to permanently embolize varicoceles b/c associated with infertility
AVM’s what is true?
• a) embolize feeding artery peripherally
• b) pulmonary AVM need to coil
• b) pulmonary AVM need to coil
What is false regarding angiodysplasia?
• a) can see bleed during angiography
• b) large draining vein
• c) usually right side
) can see bleed during angiography
A pt gets a percutaneous gastrostomy in IR. On the CXR the next day, free air is noted under the diaphragm. Choose the best answer.
• This is a normal finding in the first 24-48 hrs, and you should continue with fluids (not food) thru the PEG for a day or so.
• Pt should undergo removal of the PEG and laproscopic placement of a new PEG by surgery.
• Check the position of the side-holes.
• Place PEG to wall suction.
• Replace PEG with one of 1-2 French larger diameter.
• This is a normal finding in the first 24-48 hrs, and you should continue with fluids (not food) thru the PEG for a day or so.
Which is TRUE regarding TIPS:
• should be done for prevention of bleeding varices or failed esophageal sclerotherapy.
• Should be done for treatment of ascites in pt NOT undergoing medical tx for said ascites.
• Should be done for DEFINITIVE tx of ascites
• Should be done for tx of hepatic encephalopathy.
• should be done for prevention of bleeding varices or failed esophageal sclerotherapy.
A patient that has just undergone right lower lobe lung biopsy begins to cough up blood. What is the next step?
• A. Put patient in left lateral decubitus position
• B. Intubate the right mainstem bronchus
• C. Put patient in right lateral decubitus position
• D. Bronchoscopy
• C. Put patient in right lateral decubitus position
May-Thurner syndrome-
Patient with swelling of left leg- compression of left vein by right iliac artery
If a patient is referred for nephrostomy placement in the setting of obstructing ureteral stone with pyleo/pyonephrosis:
• you should do the procedure but warn the referring clinician that the pt’s sepsis may initially worsen.
• Attempt to disempact the stone
• Refuse the procedure.
• Upon entering the calyx, immediately distend the renal pelvis with contrast.
• Enter the anterior calyx, thus avoiding traversing much of the renal parenchyma with the needle (something similar to this).
• you should do the procedure but warn the referring clinician that the pt’s sepsis may initially worsen.
Best placement for a nephrostomy tube is:
• posterior calyx
• anterior calyx
• infundibulum
• something about lateral renal cortex (????)
• corticomedullary junction
• Post calyx
Second most common origin of the left vertebral artery is:
• A. Off aortic arch between the left subclavian and left carotid
• B. Off of left coronary
• C. Off aortic arch distal to subclavian artery
• D. Off of thyrocervical artery
• E. Off aortic arch proximal to left carotid
• A. Off aortic arch between the left subclavian and left carotid
• What do the vasa vasorum do?
• Vasa vasorum means vessel within a vessel. In large and medium sized arteries, the cell layers of the media near the lumen are fed by diffusion. Because diffusion of oxygen from the lumen is inadequate to supply the outer cell layers of the media, arterioles arising from the adventitia send vascular twigs to the outer one-half to two-thirds of the media. These vessels are called the vasa vasorum. Interruption of flow in the vasa vasorum may lead to medial necrosis and aneurysm formation.
• Much basic science is being performed on the effects on the vasa vasorum from angioplasty and stent placement. Stents and angioplasty may lead to a temporary proliferation of the vasa vasorum.
• Sources: 5th edition of Robbins Pathologic Basis of Disease page 468
• Cragg et al. The vasa vasorum and angioplasty. Radiology, 1983;148:75-80
• Pisco et al. Vasa vasorum changes following stent placement in experimental arterial stenoses. JVIR 1993;4:269-73
• Regarding splenic artery aneurysms:
• A. More common in men – Definitely wrong. Splenic artery aneurysms occur with a 4:1 female preponderance.
• B. Rupture in pregnancy – Best answer. Of more than 400 cases of ruptured splenic artery aneurysms in the literature, about 100 cases of ruptured splenic artery aneurysm during pregnancy have been reported, with only 12 cases of maternal and fetal survival.[42] Rupture occurred during the third trimester in two thirds of the cases and was typically misdiagnosed as splenic rupture or uterine rupture. Maternal mortality was 75% with a fetal mortality of 95%. Increased portal pressures, high splenic artery flow due to distal aortic compression, and progressive arterial wall weakening are contributing factors. Multiparity may increase the risk; 78% of patients with ruptured splenic artery aneurysms have been in their third pregnancy. Survival is most likely related to a “two-stage rupture,” in which the lesser sac temporarily tamponades the bleeding aneurysm. Townsend: Sabiston Textbook of Surgery, 17th ed
• C. Present with left upper quadrant mass – Pretty much wrong. The majority of patients are asymptomatic, although epigastric or left upper quadrant pain may occur. Messina and Shanely, Visceral Artery Aneurysms. Surgical Clinics of North America, vol. 77 #2 April 1997
• D. Rarely calcify – Wrong. Splenic artery aneurysms (SAAs) are most commonly identified on x-ray film by the presence of concentric calcifications in the left-upper abdominal quadrant. Splenic artery tortuosity simulating a splenic artery aneurysm. Majeski J. South Med J. 1998 Oct;91(10):949-51.
• E. Highly associated with PAN – Not quite right. PAN (Polyarteritis Nodosa)- findings include luminal irregularities, occlusive lesions/stenosis (most common) and aneurysms. Affected areas tend to be visceral arteries, extremities and small branches of aorta. Not specifically associated
with splenic artery. Also found in many diseases, not just PAN, including pancreatitis, pregnancy, portal hypertension, liver transplantation. Stanson et al PAN: Spectrum of Angiographic Findings RadioGraphics 2001; 21: 151.
• Best answer: B
• A patient is s/p vascular endograft placement for and abdominal aortic aneurysm. Which of the following conditions requires immediate corrective surgery?
a. distal endoleak (Type 1A)
b. proximal endoleak (Type 1B)
c. Aorto-ilial endoleak (Type II)
d. Type III
e. Leak through porous graft material (Type IV)
Ans: A and B (any type 1 needs repair)
• Type 1 occurs at the graft insertion site due to an inadequate seal between the stent graft and the aortic wall. These leaks result in elevated sac pressure and a continued risk for rupture, requiring immediate surgery.
• Type 2 occurs when there is retrograde inflow in a patent branch vessel. Retrograde endoleaks are unavoidable with current endovascular techniques [8]. The risk for type 2 endoleak is increased with increasing patent side branch vessels [7]. The clinical importance of Type 2 endoleaks is not clear. Small leaks can be associated with a stable sac size and can spontaneously resolve. However, this type of leak (even small leaks) is often associated with failure of the aneurysm to decrease in size and it may actually increase.
• Type 3 occur due to graft degeneration including component disconnection, fabric tears, and disintigrationl
• Type 4 leaks occur due to transgraft flow due to graft wall porosity
What is blue toe syndrome?
• DVT
• Atherosclerosis
• Dissection
• Atheroembolism
Ans: D.
 The blue toe syndrome is characterized by tissue ischemia secondary to cholesterol crystal or atherothrombotic embolization leading to occlusion of small vessels. Embolization occurs typically from an ulcerated atherosclerotic plaque located in the aorto-iliac-femoral arterial system. Clinical presentation can range from a cyanotic toe to a diffuse multiorgan systemic disease that can mimic other systemic illness. Mortality can be higher than 70% depending on the scope of the illness. Embolization can occur spontaneously or from a variety of insults such as invasive vascular procedures, anticoagulation, or thrombolytic therapy. Angiography, duplex ultrasonography, computerized tomographic scanning, and magnetic resonance imaging have been used to image the offending lesions, with angiography considered the "gold standard" despite its inherent risks.
Curr Opin Cardiol 1996; 11:533-42. (AN 97044310; PMID 8889381) Cath exchange brady/hypoxic
When placing a new indwelling SVC catheter, the patient becomes bradycardic and hypoxic immediately after placing the catheter in the peel-away sheath. Most likely cause?
• vasovagal reaction
• air embolism
• Caval perforation
B air embolism
• Which of the following is the most vasoconstrictive substance?
a. Angiotensinogen
b. angiotensin I
c. angiotensin II
d. renin
C. Angiotensin II
• A patient with a Mallory-Weiss tear and UGI bleeding is sent for angiogram. What is the most likely vessel bleeding?
a. Hepatic artery
b. Adrenal artery
c. Inferior phrenic artery
d. Left gastric artery
e. Inferior esophageal artery
f. Gastroduodenal artery
g. Coronary vein
h. Azygous vein
Answer: D. Left gastric artery (branch of).
• Definitely, the most common cause of persistant bleeding (failure conserve therapy) is arterial. This is where VIR comes in and embolizes left gastric artery branch. Gelfoam (temporary) or vasopressin typically used. Gore/Levin GI radiology (p 512, p 2532) and B&H (p 600) both discuss arterial based bleeds assoc with M-W tears and the appropriate arterial catheter-based therapy. However, majority of m-w tears stop bleeding spon w/o therapy which seems to imply venous source majority of time. Dahnert says involvement of venous plexus. I will prob go with arterial if given answer choices of left gastric artery vs coronary vein. Pruitt says arterial . Dolmatch says arterial.
• What is the origin of the uterine artery?
a. external iliac a.
b. anterior division of internal iliac a.
c. posterior division of internal iliac a.
d. Aorta
b. anterior division of internal iliac a.
Which of the following is the most common renal vascular anatomy (may have been IVC anatomy)?
• Circumaortic left renal vein
• Retroaortic renal vein
• Duplicated IVC
• azygous continuation of the IVC
left IVC
A circumaortic
 In about 2% of cases (1.8-2.4% of all abdominal CT scans), the left renal vein may be retroaortic or even circumaortic; either single vessel or doubled with one limb anterior and the other posterior to the aorta reported frequency 1.5-4.4% of all abdominal CT scans), or it may form a circumaortic renal venous collar (reported frequency 1.5-8.7%).
 4–16% of cases the renal vein is circumaortic with two renal veins, one anterior (preaortic) and the second posterior (retroaortic
 Seen in up to 8-9% of the population vs.
 2% having duplicated IVCc.
• Most likely anatomically significant IVC anomaly encountered in IVC filter placement:
• a) retroaortic left renal vein
• b) circumaortic left renal vein
• c) left IVC
• d) azygous continuation of the IVC
Maybe should be b, circumaortic.
• Duplication of the IVC
• Duplication of the IVC is observed in 0.2-3% of individuals. When this anomaly is present, the left IVC usually is smaller than the right, although they can be equal in size. The left IVC typically empties into the left renal vein and, subsequently, into the right IVC. Although the frequency of duplication is low, demonstration of the anomaly is important when determining the position for IVC filter placement. In the presence of a duplicated IVC, filter placement in the right IVC may not be adequate to prevent PE, especially in the presence of a left lower-extremity DVT. Placement of a filter in both the left and right IVCs is often required for true PE prophylaxis in the presence of a duplicated IVC.
• Transposition of the IVC
• Transposition of the IVC is observed in 0.2-0.5% of individuals. A left-sided IVC drains into the left renal vein and crosses to the right of the spine. It then continues cranially in the normal position. The left renal and suprarenal veins empty directly into the left IVC, while the right gonadal and suprarenal veins drain into the right renal vein and, subsequently, into the normal right-sided prerenal division of the left-sided IVC.
• Circumaortic and retroaortic left renal vein
• The incidence of circumaortic renal veins has been reported to be as high as 8.7%. Patients with circumaortic or multiple renal veins may have a large hilar communication, which provides an alternative conduit for emboli. In the presence of a circumaortic renal vein, a filter immediately inferior to the main left renal vein may not represent adequate prophylaxis because of the alternative conduit; therefore, filters should be positioned inferior to the circumaortic renal vein. A retroaortic left renal vein is more common but does not impact filter placement because it does not represent a potential conduit for thrombus arising from the lower extremities.
• The most common anomaly of the left renal venous system is the circumaortic renal vein, seen in up to 17% of patients (21). In this anomaly, the left renal vein bifurcates into ventral and dorsal limbs that encircle the abdominal aorta. The posterior limb is usually the smaller of the two, although this is certainly variable
• 49. A patient presents with a normal single right renal vein and circumaortic left renal vein. If you have to place an IVC filter in this patient, where would you place it?
• About the three renal veins
• Below the right renal vein
• Below the retroaortic left renal vein
• Place a filter in the IVC and retroaortic vein
C Below the retroaortic left renal vein

• Filters are best placed in the IVC immediately below the most inferior renal vein origin to provide continuous flow across the dome of the filter. The posterior half of the left renal joins the IVC at a lower level than the anterior half. (UCSF teaching files)
Patient with prosthetic mitral valve is to have percutaneous biliary drainage. What’s the best management?
• Premedicate with IV Ampicillin and Gentamicin, then give PO Amoxicillin q8h x2
• Premed with IV Vancomycin, then PO Amoxicillin q8h x2
• Only give PO Amoxicillin q8h x2 (after procedure)
• Premed with IV Ceftriaxone, then PO Amoxicillin q8h x2
• No antibiotics necessary, as the bile is sterile
• Premedicate with IV Ampicillin and Gentamicin, then give PO Amoxicillin q8h x2

 Standard antibiotic treatment for high-risk patient with prosthetic heart valve undergoing GU or GI procedure is:
 Amp 2.0g IV (or IM) + Gent 1.5mg/kg IV (or IM) not to exceed 120mg 30 minutes prior to procedure.
 Amox 1g PO/IV/IM 6 hours after first dose.
 Standard treatment for moderate risk patient with heart valve is:
 Amox 3.0g PO, AMP 2.0g IV OR
 Vanc 1.0g IV 30 minutes before procedure
 From Textbook of cardiovascular medicine Topol et. Al.
 Standard general prophylaxis: amoxicillin 2.0 g orally (po) (adult) or 50 mg/kg po (child), 1 hour prior to the procedure. Alternative for those unable to take po is ampicillin 2.0 g intravenously or intramuscularly (IV/IM) (adult) or 50 mg/kg IV/IM (child), within 30 min before procedure
• Indications for PTC include all the following except:
• a) pruritis
• b) stone removal
• c) acute cholecystitis
• d) cholangitis
• e) bile leak
Answer: C acute cholecystitis

• Stenting may relieve pruritus and improve quality of life.
• Percutaneous Transhepatic
• Cholangiography: Indications (1– 6)
• 1. Define level of obstruction in
• patients with dilated bile ducts
• 2. Evaluate for presence of suspected
• bile duct stones
• 3. Determine etiology of cholangitis
• 4. Evaluate suspected bile duct
• inflammatory disorders
• 5. Demonstrate site of bile duct leak
• Table 2
• Percutaneous Transhepatic Biliary
• Drainage: Indications (7–10)
• 1. Decompress obstructed biliary tree
• 2. Dilate biliary strictures
• 3. Remove bile duct stones
• 4. Divert bile from and stent bile duct
• regarding variceles, which is true?
– Due to the lack of collaterals, angio embo has a technical success rate of close to 100%
– Surgical and angiographic treatments have similar clinical outcomes
• A. angio works well.
• what is a contraindication to TIPS
– Right heart failure
– Left heart failure
Right heart failure

Absolute contraindications include the following:
• Right-sided heart failure with increased central venous pressure
• Polycystic liver disease
• Severe hepatic failure
Relative contraindications include the following:
• Active intrahepatic or systemic infection (Bacteria can colonize the stent, causing persistent infection.)
Which of the following should be the end result of a TIPS procedure:
• a) left hepatic vein to left portal vein branch anastamosis
• b) gradient > 15 mm HG
• c) hepatopetal flow in the main portal vein after the procedure
• d) TIPS is indicated for isolated gastric varices
Answer: C
read carefully. Depends on where you look.
• Normal shunt has:
• Hepatofugal flow within the intrahepatic
• right and left portal veins beyond the site
• of the connection between the portal vein
• and the TIPS

• Main portal vein would have hepatopedal as it flows toward tips shunt.

• Helical or Hepatofugal flow in the
• portal vein branches is frequently seen af-
• ter successful TIPS and reflects complete
• diversion of portal venous flow from the
• hepatic parenchyma into the shunt. Al-
• teration in flow direction in a portal vein
• branch from hepatofugal to hepatopetal
• may indicate stent malfunction
• where is the most common site of complication for a dialysis graft?
– Venous end
• Which of the following is true regarding aortic transection?
a. death is usually from aortic insufficiency
b. if untreated most result in death
c. dissection seen in 50%
d. most common location takeoff of the inominant
• B. if untreated most result in death
• Which of the following is the most common inheritable hypercoagulable disorder?
a. Sickle cell disease
b. Protein C / S deficiency
c. Factor V Leiden
• C. Factor V Leiden
• Initial angiogram imaging to evaluate for popliteal artery entrapment are negative. What maneuver should be performed next?
a. Inflate blood pressure cuff on the calf
b. Knee flexion
c. Flexion at hip
d. Images of the foot in plantar flexion
e. Raise leg
• D. Images of the foot in plantar flexion. Represents compression of the popliteal artery either by abnormal medial course of the popliteal artery in relation to the medial head of the gastrocnemius muscle, an abnormal insertion of the muscle itself [e.g., aberrant slip of muscle] or compression by the muscle. Bilateral disease is seen in 20%, with males more commonly affected (8:1). Clinical presentation is unilateral calf claudication in a young male, which may be sudden. Classic angiographic findings include medial deviation of the popliteal artery and stenosis. Stress views (active plantar flexion against resistance) may be necessary to evoke the findings (active plantar flexion will obliterate the tibial pulse as well). Other findings include popliteal artery thombosis and aneurysm formation. (common repeat, Main Source: Dähnert 5th p. 643)
• Which of the following is NOT an indication for emergent placement of percutaneous nephrostomy tube in a patient with hydronephrosis?
a. hyperkalemia
b. Congestive Heart Failure (with hydronephrosis)
c. Urosepsis
d. Urinary encephalopathy
e. Pyonephrosis
f. Anuria
• B. Congestive Heart Failure (with hydronephrosis):
• Indications for percutaneous nephrostomy: 1. Pyeloureteral obstruction causing hydro- or pyonephrosis. 2. Diversion of urine form renal collecting system in order to heal leaks,/ fistulas. 3. Decompression of renal/perirenal fluid collections. 4. Tract creation for stone retrieval/biopsy/stricture dilatation/antegrade ureteral stent. 5. Direct infusion of drugs (dissolving stones / antitumor / antibacterial / antifungal). 6. To relieve urinary tract obstruction of pregnancy. (*Interventional radiologists have indicated the metabolic derangement secondary to acute renal failure.)Complications occur with 2-4%. Complications include sepsis, hemorrhage and adjacent organ injury. (Old question 1999, Primer 259). The only real contraindications are a bleeding diathesis (most commonly uncontrollable coagulopathy) and an uncooperative patient. Severe hyperkalemia (> 7 mEq/L) should be corrected with hemodialysis prior to the procedure. www.emedicine.com
• Which of the following is false regarding venous thoracic outlet syndrome?
a. thrombolysis is an accepted treatment
b. results from occlusion of a vein
c. commonly occurs with exercise
d. most commonly seen in elderly females
• D. Most commonly seen in elderly females. Thoracic outlet syndrome be caused by compression / occlusion of the nerves (> 95% of cases), arteries, or veins (2-3%) at the thoracic outlet. Primer 3rd. p. 700.
• Subclavian vein thrombosis (as specified in this question) is also referred to as thoracic outlet thrombosis or effort thrombosis (= Paget-Schroetter disease). The thrombosis is due to chronic injury to the vein as it passes over the first rib behind the anterior scalenus muscle, where it is compressed by the clavicular bone and by the costoclavicular ligament. The subclavius muscle may also play a role in its pathogenesis. The accepted treatment regimen has been anticoagulation treatment or fibrinolytic treatment. (17-75% disability w/o treatment). There is consensus among surgeons that those patients with thoracic outlet syndrome and thrombosis should have the first rib resected and the anterior scalenus muscle removed after successful thrombolysis. Castañeda synopsis p. 200The disease is more commonly associated with younger, active people; thus choice ‘D’ is untrue (and the correct answer).
patient who has been using crutches presents with a warm extremity, blue fingertips, normal distal pulses, and bounding axillary pulses. The most likely diagnosis is:
a. thrombosed axillary a.
b. pseudoaneurysm of axillary a.
c. cervical spine foramen stenosis
d. Reflex Sympathetic Dystrophy
e. Brachial Plexus Vasomotor instability
• B. pseudoaneurysm of axillary
• Crutch trauma accounts for most of the cases of axillary aneurysms (54%). The normal distal pulse rules-out a thrombosed axillary artery, and the blue fingertips are from microemboli (formed in the pseudoaneurysm). [see Feldman DR, et al. Crutch-induced axillary artery injury. Cardiovasc Intervent Radiol. 1995 Sep-Oct;18(5):296-9]Pseudoaneurysms normally arise from a full thickness focal defect in the arterial wall, that lead to a saccular dilatation. In cases where the hemorrhage is contained by surrounding tissues, the hematoma may continue to communicate with the bloodstream through the arterial wall defect. The hematoma may then transform into an aneurysm sac by way of lysis or from compression of its contents by the pulsating stream that communicates with it . www.emedicine.com
• What is May-Thurner syndrome?
• B. Compression of the left common iliac vein by right common iliac artery.
• May-Thurner Syndrome is compression of the left common iliac vein by the anteriorly crossing right common iliac artery. The constant, pulsatile compression is thought to induce intimal scarring and weblike adhesions within the vein, resulting in chronic left leg swelling and risk of venous thrombosis. Similar upper extremity syndrome: Paget-Schroetter Sydrome – Primary axillosublcavian thrombosis. Precipitated by extrinsic subclavian vein narrowing resulting from normal variations in anatomy such as cervical rib, slips of fascia, or hypertrophied musculature. (Brand and Helms p609 and 623)
• An elderly man presents with acute onset of back pain, blood pressure of 100/60, heart rate=135, and a palpable pulsatile abdominal mass. The most appropriate initial imaging:
a. Doppler ultrasound
b. Angiography
c. CT without contrast
d. MRI with and without gadolinium
• C. CT without contrast Pt likely has an abdominal aortic aneurysm. Evaluation with CT is preferred. Lack of contrast will still show acute rupture or leak and increase through time. Incidence: 5-7% of over 60 years old, over 60 years accounts for 75% of aneurysms. Male:Female Ratio: age: 60-64 year old: 11:1, age: 85-90 year old: 3:1 Risk Factors: Tobacco abuse, First degree relative with AAA , Atherosclerosis, Hypertension, Chronic Obstructive Pulmonary Disease, Increasing age Male sex Causes :Associated with Atherosclerosis in only 25% of patients, Aortic Dissection, Mycotic Infection, Cystic Medial Necrosis, Ehlers-Danlos Syndrome
• Symptoms: Pain in Abdomen, Flank or Back, Early satiety, Nausea and Vomiting, Pulsatile Abdominal Mass >30 mm
• Radiology: Standard Abdominal CT: Indications-Suboptimal Ultrasound of AAA, Suspected hemodynamically stable, ruptured AAA (Brandt and Helms 577)
• Which is true regarding renal artery angioplasty?
a. highest success rate is with ostial lesions
b. hypotension is a complication
c. non lateralization of renin levels is a contraindication
d. hypertension improves in 90% of cases
• D. hypertension improves in 90% of cases:
• In the Univ. of Virg. Series, 94% of 65 HTN patients with atherosclerotic lesions were helped by the procedure. For patients with FMD, “…90% will improve.” Castañeda synopsis p. 200
• Recognized complications of renal angioplasty (in 5-10%):
renal insufficiency (contrast); subintimal dissection; renal artery thrombosis; renal artery rupture; distal embolization; renal artery spasm; and hematomas at the puncture site. Note that while hypotension may certainly occur post-procedure [it is recommended that the patient be observed for 48 hours because “drastic blood pressure changes may occur”] this is not an accepted “complication”. Castañeda synopsis p. 200
• Success is much lower in true ostial lesions. Initial technical success rate is 80-90%. Therapeutic success (favorable BP results is also high, especially in FMD (90-100%) but somewhat lower in atherosclerotic disease (70-90%). (Primer 606, 636 and Old questions). Note that while renal vein renin sampling might be helpful, it is only 75-80% sensitive [fails to lateralize in 20-25%]. [InterNet]
• What is the origin of the uterine artery?
a. external iliac a.
b. anterior division of internal iliac a.
c. posterior division of internal iliac a.
d. aorta
• B. The anterior division of the internal iliac artery. This vessel usually arises separately from the internal iliac artery, but it may arise from the umbilical artery. It descends on the lateral wall of the pelvis, anterior to the internal iliac artery, and enters the root of the broad ligament where it passes superior to the lateral portion of the fornix of the vagina to reach the lateral margin of the uterus. The uterine artery passes anterior and superior to the ureter near the lateral portion of the fornix. Clemente 4th Plate 265.The fact that the uterine artery crosses anterior and superior to the ureter near the lateral portion of the fornix of the vagina is clinically important. The ureter is in danger of being inadvertently clamped or severed during a hysterectomy when the uterine artery is tied off. The point of crossing of the uterine artery and ureter lies about 2 cm superior to the ischial spine. The left ureter is particularly vulnerable because it is very close to the lateral aspect of the cervix. The ureter is also vulnerable to injury when the ovarian vessels are being tied off because these structures lie very close to each other where they cross the pelvic brim. Moore 264,Important cross question: Testicular vein is anterior to the ureter. Branches of the IMA: left colic, sigmoid, superior hemorrhoidal artery.
• Failure of gortex dialysis graft is most likely secondary to which of the following?
a. Central venous stenosis
b. arterial anastomosis stenosis
c. venous anastomosis stenosis
d. Intimal hyperplasia
• C. Venous anastomosis stenosis: Stenosis of a graft most commonly occurs at the venous anastomosis (in 80%). Failure of a native AV fistula is more likely to be due to stenosis either centrally or at the arterial anastomosis. Castañeda synopsis p. 120 I think that the question or answered may have been answered incorrectly or in true ABR fashion poorly worded. In Initimal hyperplasia: the tissue lining the vein may be growing faster which leads to narrowing a.k.a. stenosis. In essence, the graft leads to intimal hyperplasia which can lead to stenosis. Venous stenosis is the best answer—see Kandarpa 2nd p. 120.
• Where is the most common location of extracranial carotid atherosclerotic disease?
a. Common carotid a.
b. External carotid a.
c. Internal carotid a. near bulb
d. Distal extracranial internal carotid a.
• C. Internal carotid a. near bulb:
• Carotid plaques form preferentially at the origin of the ICA (~34%). The theory is that altered wall shear stress in an area of flow separation (i.e., the carotid bifurcation) favors plaque formation. Dähnert 5th p. 371-373Carotid atherosclerosis causes embolic ischemia; intracranial atherosclerosis causes in situ thrombotic or distal embolic ischemia. Location: ICA origin > distal basilar > carotid siphon, MCA. Critical carotid stenosis is defined as a stenoiss of > 70% in luminal diameter. Patients with critical stenosis and symptoms have an increased risk of stroke and benefit from carotid endartectomy. Patients with stenosis < 70% or who are symptomatic are usually treated medically: Primer 472
• When consenting a patient for placement of a chest wall port, which of the following does not need to be mentioned during the consent?
a. “pinch off”
b. infection
c. malposition of catheter tip
d. thrombosis of catheter
e. bleeding
• A. “pinch off” Pinch off syndrome is not considered a technical complication as it can happen later, not a result of insertion. It may lead to thrombosis or emboli. This “syndrome” arises when a subclavian catheter passes through a confined anatomic space and becomes compressed or kinked. Most often this is positional i.e. better or worse in certain positions. In fact, with pinch off syndrome the catheter may not be able to infuse or withdrawn until the patient lies down or rolls the shoulders back. Pinch off may lead to catheter malfunction, catheter extravasation or at worst – catheter fragmentation and embolism. The average indwelling catheter duration until fragmentation approximates 6 months. The confined space is called the costoclavicular triangle or space.
Medial insertion of catheter [L], correct insertion [R] Pinch off syndrome can easily be prevented by using the internal jugular approach. If no other option is available it is best to use an axillary approach as opposed to the more traditional medial approach. This ensures that the catheter is within the subclavian vein as it passes through the costoclavicular triangle. Therefore, it can not be compressed (see above). www.venousaccess.com
• Which of the following is false about varicose veins?
a. DVT causes valve dysfunction 90% of the time
b. reflux is best evaluated standing upright with ultrasound
c. valves are always incompetent
d. may be caused be small perforating vessels
e. a permanent material should be used for embolization
• A. DVT causes valve dysfunction 90% of the time. Some experts believe that valve incompetence is the cause of varicose veins. It is theorized that valve failure at the saphenofemoral junction permits reflux into the saphenous vein, resulting in descending sequential valvular incompetence from the thigh to the calf. A contrasting theory is that one or more perforator veins in the lower leg result in high-pressure flow and increased volume from deep to superficial veins during muscular contraction. In time, superficial veins become dilated, separation of the valve cusps prevents their apposition, and flow reverses in the affected veins. As other perforator valves become incompetent, reflux occurs at additional sites. Progression of these factors proximally in the long saphenous vein causes secondary incompetence at the saphenofemoral junction. This theory explains the initial occurrence of varicose veins and the development of new ones after any form of treatment but does not explain why veins used as arterial bypass conduits (at arterial pressures) do not become varicose.
• In what aspect is angiography better than MRI for a Type A aortic dissection?
a. pericardial tamponade
b. site of intimal flap
c. seeing true and false lumen
d. visualization of coronary arteries
e. Aortic insufficiency
• D. Involvement of coronary arteries Angiography (86-88% sensitive) is superior to any other technique for demonstrating:
- entry + reentry points
- branch vessel involvement + coronary arteries
- aortic insufficiency Dähnert 5th p. 609MRI is extremely accurate with sensitivities and specificities exceeding 90%. CT and MR are primary imaging modalities in the postoperative patient and in the evaluation of chronic dissection (10%). Routine spin-echo imaging will demonstrate findings similar to those obtained on CT. The intimal flap will be outlined by “dark blood”, where as GRE MR will delineate the flap surrounded by “bright blood”. Brandt and Helms 583Aortography is the procedure of choice in the evaluation of acute dissection. Its importance in preoperative planning is to demonstrate the extent of the dissection, site of intimal tears, aortic valve regurgitation, coronary artery involvement, and filling of branch vessels. The classic finding (87%) is a “double barrel” aorta with an interposed intimal flap. The intimal flap usually begins in the right anterolateral ascending aorta and spirals to the left posterolateral aspect of the descending aorta into the abdomen. The left renal artery is frequently supplied by the false lumen, and the left iliac artery is more commonly involved when the dissection extends distally.
• A male presents with a pulsatile groin mass one week after undergoing cardiac catheterization. What is the most appropriate next step in management?
a. Perform ultrasound and consider thrombin injection
b. CT
c. Repeat catheterization / Angiography
d. MRI
e. Refer to surgery
• A. Perform ultrasound and consider thrombin injection: Most complications of angioplasty can be managed nonoperatively. The most common complication is a hematoma, usually self-limited at the puncture site. Pseudoaneurysms can be treated definitively by using ultrasound-guided compression or percutaneous thrombin injection. Vascular and Interventional Radiology: Principals and Practice 76In situ thrombosis or distal emboliztion during or after angioplasty can be treated with intra-arterial thrombolytics, with suction thromboembolectomy using a guided catheter or with a “Possis Angiojet device”.
• Which of the following is the most important factor for developing a pneumothorax during a percutaneous lung biopsy?
a. needle size
b. tumor size
c. COPD
d.
• C. COPD
• There are two common and important complications of lung biopsy: hemoptysis and pneumothorax. Hemoptysis, to some degree, is noted in up to 20% of patients and is usually benign and self limited. Severe hemoptysis is requiring intervention is unusual. Pneumothorax is observed in 5-30% of patients. Risk factors include COPD, biopsies at the lung apicies, deep lesions and transgressions of pleural surfaces located along the fissures. Most pneumothoraces do not require a chest tube (biopsied induced). Interventional Radiology EssentialsVascular and Interventional Radiology 114: Increased lesion depth, pre-existing lung disease (COPD) and small lesion size have been shown to add to development of pneumothorax.Therefore: pneumothorax risk factors are: COPD, apical lesions, depth, number of pleural surfaces violated.
• Being stuck by which of the following will result in the greatest risk to a health care worker for contracting HIV?
a. Bloody scalpel
b. Bloody hollow needle
c. Tears / urine
d. Solid point bloody needle
e. Prolonged exposure of blood on skin
• B. Bloody hollow needle:The overall risk of HIV seroconversion with a contaminated needle stick is < 1% (CDC Web Site).The percutaneous exposures most frequently involved hollow-bore and solid needlestick injuries; a few involved other sharp objects.
Various factors increase the risk of acquiring HIV infection. These include: (i) The depth of the injury (in case of a sharp object), (ii) Whether the device was visibly contaminated with blood, (iii) Whether the procedure involved placing a needle directly in an artery or vein, (iv) Whether the needle was a hollow bore needle or a solid needle (e.g. suture needle), (v) the size of the needle (large versus small gauge), (vi) the patient's viral load (i.e. amount of HIV in circulation), (vii) the amount of blood or infectious fluid involved in the exposure, and (viii) the duration of the exposure.Blood and bloody fluids are considered infectious. Potentially infectious materials include semen, vaginal secretions, CSF, pleural, peritoneal, pericardial, amniotic fluids or tissue. Exposure to saliva, tears, sweat, non-bloody urine or feces is not believed to pose a risk. www.cipladoc.com
• Which of the following is true about type A aortic dissection?
a. Non-displaced NG tube excludes
b. Other answers not recalled
• Type A dissection (60%) involves the ascending aorta, with variable involvement of the arch and descending aorta. Treatment is surgical. Type B (40%) is limited to the arch and descending aorta, similar to Debakey type III. Treatment is medical or surgical.Debakey I (30%) involves the ascending aorta, arch, and variable portion of the descending aorta (tx – surgical). Debakey II (20%) is limited to the ascending aorta and carries the worst prognosis (tx – surgical). Debakey III (50%) originates near the isthmus distal to the left sublcavian artery origin. This lesion carries the best prognosis. Tx is medical. No chest findings are specific or sensitive. A non displace NG tube does not exclude a dissection. Recent question indicate that displacement (of the left mainstem bronchus) is often noted. Mediastinal widening is the most common chest finding (80%) followed by a double aortic contour (double aortic knob sign) 40%. All chest findings require further workup. Brant and Helms 581-3, Extensive discussion in Dahnert 509.
• Which of the following is false about abdominal aortic aneurysms?
a. 90% are infra-renal
b. commonly extends to involve iliac arteries
c. associated with popliteal aneurysm
d. inferior mesenteric artery is occluded in most
e. perforation risk of a 4 cm aneurysm is greater than surgery
• E. perforation risk of a 4 cm aneurysm is greater than surgery:
• Location: infrarenal (91-95%) with extension into iliac arteries (66-70%). Surgery recommended if > 5 cm in diameter. Dähnert 5th p. 605.Aneurysm Rupture: Incidence < 4cm in 10%; 4-5 cm in 23%, 5-7% in 25%, 7-10 cm in 46%, >10 cm in 60%. Treatment 4-5% surgical mortality for non ruptured, 30-80% for ruptured aneurysm.
Prognosis 17% 5 year without surgery, 50-60% with surgery. Associated with common iliac artery aneurysm in 89% and internal iliac 10%.50% of popliteal artery aneurysms are associated with an AAA [InterNet]; associated with IMA occlusion in 80% Dähnert 5th p. 605.
• What is the origin of the left bronchial artery?
a. pulmonary artery
b. intercostals artery
c. thoracic aorta
d. subclavian artery
• C. thoracic aorta The bronchial arteries supply blood to the connective tissue of the bronchial tree. The two left bronchial arteries arise from the superior part of the thoracic aorta, superior and inferior to the left main bronchus. The single right bronchial artery often arises as a common trunk with the third or fifth posterior intercostals artery or from the superior left bronchial artery. Moore 76, 115, 118
• Which of the following reverses the effects of benzodiazepines?
a. Naloxone
b. Flumazenil
• B. Flumazenil Interventional Radiology Essentials 251. Reversal Agents:Flumazenil (Mazicon) is an agent developed to reverse the sedative and respiratory depressive effects of benzodiazepines. The usual dose is 1 mg over to 1 to 3 minutes up to 5 mg in a 10 – minute period. Seizures can be precipitated in patients who are habituated to benzodiazepines.The respiratory depression caused by overadministration of narcotic analgesics can be reversed with naloxone (Narcan). The dosage is .05 mg to 0.2 mg given IV every 2 to 3 minutes. Rebound hypertension, agitation, and tachycardia may be observed. Congestive heart failure may also be precipitated by diminished venous compliance. IN patients who have received both opiates and benzodiazepines, the opiates should be reversed first.
• Which of the following is false regarding AVM’s?
a. the goal of therapy is to embolize the feeding vessels
b. coils should be used for pulmonary AVM’s
c. same rate of recurrence as systemic AVM
d. hemoptysis is a rare presentation
e. pulmonary infarct is not the most common complication of embolization
C,d, and e are false not same rate of recurrence. Dahnert p429
• Answers and/or questions probably remembered incorrectly:Answer choice a and b appear correct.My notes show that another answer choice was The feeding arteries and draining veins should be occluded,which I believe to be the false answer. Old tests show that this question this has been asked in one form or another several times (98, 95, 92, etc).Some basic facts to know from previous T / F questions: Permanent agents, such as coils or balloons, not gelfoam or Ivalon, are the agents of choice for embolization. Systemic complications of embolization occur more frequently than pulmonary infarcts. Hemoptysis, cyanosis, clubbing, and bleeding from other sites are common complications of pulmonary avms. 40-65% of pulmonary AVMs are associated with Osler-Weber_Rendu (autosomal dominant). AVMs occur most commonly in the lower lobes and are twice as common in women. They occur most often in the third and fourth decade They are defects in the terminal capillary loops of the pulmonary arteries. They are not supplied by the bronchial arteries. Dähnert 429 – long list of facts.Good quick review: Abnormal communication b/w pulmonary artery and veins. Types: congenital (60%) Osler-Weber_Rendu disease (hereditary hemorrhagic telangiectasia). Acquired (40%): iatrogenic, infection, tumor. Radiograpic pearls: lower lobes 70% > middle > upper. Feeding artery with feeding veins. Sharply defined mass, strong enhancement, change in size with Valsava / Mueller movement. Primer 63-64
• Which of the following is true regarding splenic artery aneurysm?
a. rarely calcify
b. Most often ruptures during pregnancy
c. presents as pulsatile mass
d. males > females
e. polyarteritis nodosa a common cause
. Most often ruptures during pregnancy
• Splenic artery aneurysms account for 60% of visceral artery aneurysms. Most are solitary (70%) and located in the distal third of the artery (80%). Patients are usually asymptomatic. They are most common in women (4:1) and have an increased rupture rate of pregnancy. Rupture has mortality of 25% but occurs in only 2% of cases not associated with pregnancy. Aneurysms that are symptomatic, enlarging, associated with pregnancy or in women of child-bearing age, or are > 2 cm in diameter usually require treatment. The two-thirds of aneurysms that contain calcification have a lower risk of rupture. Brant and Helms (590)Etiology: medial degeneration with superimposed atherosclerosis, congenital, mycotic, pancreatitis, trauma, portal HTN Dähnert (537):Rupture of aneurysm (6-9%, higher during pregnancy) with up to 76% mortality. DDx: renal artery aneurysm, tortuous splenic artery.
• Patient presents with Varicocoele, what is the treatment?
a. Permanent occlusion of spermatic vein
b. Orchiectomy
c. Alcohol ablation
d. Temporary occlusion of spermatic vein with gelfoam
• A. Permanent Occlusion of Spermatic Vein.
• Varicocele represent dilated veins of the pampiniform plexus. They result from incompetent or absent valves in the spermatic vein. Occur in 15% of the adult males. Clinical Findings: infertility, pain, scrotal enlargement. Location: 90% on the left – drainage of spermatic vein into the left renal vein at a right angle, 25% are bilateral, solitary right varicocele requires exclusion of an underlying malignancy. US: shows hypoechoic veins (bag of worms), veins > 2cm, increase venous size with upright or valsalva, color Doppler within varicocele and increases with valsalva. Primer 304
• Components of Varicocele: a) internal spermatic vein (ventral location) draining testis b) vein of vas deferens (mediodorsal location) draining epididymis c)cremasteric vein (lateradorsal location) draining scrotal wall Dahnert 819Treatment:
1. Ivanissevitch procedure ~ surgery
2. Transcatheter spermatic vein occlusion: Coils are used in the treatment of testicular varicoceles (Picture of coil embolization of left internal spermatic vein). capillary beds and lead to tissue necrosis and infaction. Brand and Helms 625Temporary Agent: gelform – often used for GI bleeds
• Patient with gastric varices (but not esophageal varices), what is the most likely cause?
a. portal vein thrombosis
b. splenic vein thrombosis
c. superior mesenteric vein thrombosis
• B. Splenic Vein thrombosis
• Varices appear as smooth, lobulated filling defects resembling thickened folds. They are most common in the fundus and usually accompany esophageal varicies. Isolated gastric varicies may occur with splenic vein occlusion.
Brant and Helms 729
• Following administration of IV contrast a patient becomes hypotensive and tachycardic. Which should not be done
a. give oxygen
b. give atropine
c. give epinephrine
d. elevate the legs
e. give IV fluids
• B. Atropine
• Atropine should be given when there is hypotension with bradycardia indicating a vasovagal episode. Three types of reactions: anaphylactoid (idiosyncratic), nonidiosyncratic, and local reactions. All happen with HCOM and LOCM. Anaphylactoid include urticaria, facial and laryngeal edema, bronchospasm, hypotension – can be life threatening. Nonidiosyncratic – direct effect of contrast agent on organ, i.e. nephrotoxicity, cardiac arrhythmia, myocardial ischemia, vasovagal reaction. Local – basically extravasation into soft tissues, more commonly with ionic medium.
• Anaphylactoid – oxygen, saline, epi, benedryl, tagament, solumedrol, (be aware that the patient may be on a beta blocker),
• Tachy with Hypo: 02, elevate legs, saline – maybe adrenaline
• Brady with Hypo: O2, elevate legs, saline, atropine
• Concerning aortic dissection, which of the following vessels is least likely to be involved?
a. Left coronary artery
b. Left renal artery
c. Left common iliac artery
d. Left subclavian artery
e. Brachiocephalic artery
• A. Left Coronary Artery
• Helical flow pattern of blood starting anteriorly proceeding to the right lateral wall of the aorta to the superior and posterior walls of the arch and terminating in the the left iliac artery 80% of the time. 50% of all cases involve the left renal artery.
Source: Dähnert 5th ed. p.608
• What should be done if a patient becomes hypotensive and bradycardic after administration of i.v. contrast?
a. Give epinephrine
b. Give atropine
c. Give Benadryl
d. Start oxygen
e. Give i.v. fluids
• D. > E. > B. Oxygen then IVF then Atropine ABC’s of ACLS: Airway (oxygen), Breathing, Circulation Based on protocol you should order: Monitor vitals then elevate legs/place in Trendelenburg then secure the airway and give oxygen then administer bolus IVF then give Atropine if needed (.6-1mg iv slowly) May repeat Atropine up to 0.04mg/kg (2-3mg in adult)
• If patient was tachycardic then would use epinephrine 1:10,000 at 0.1mg iv slowly and repeat up to 1 mg total.
Source: ACR Manual on Contrast Media Ed. 4.1 p.36
• Hypotension + bradycardia = vagal reaction see Kandarpa 2nd p. 321.
• Treatment of pulmonary AVM includes:
a. Coils
b. Gelfoam
c. Ivalon particles
d. Ethyl alcohol
• A. coils.
• Some basic facts to know from previous T / F questions: Permanent agents, such as coils or balloons, not gelfoam or Ivalon, are the agents of choice for embolization for pulmonary AVM’s. Systemic complications of embolization occur more frequently than pulmonary infarcts. Hemoptysis, cyanosis, clubbing, and bleeding from other sites are common complications of pulmonary avms. 40-65% of pulmonary AVMs are associated with Osler-Weber_Rendu (autosomal dominant). AVMs occur most commonly in the lower lobes and are twice as common in women. They occur most often in the third and fourth decade They are defects in the terminal capillary loops of the pulmonary arteries. They are not supplied by the bronchial arteries. Dähnert 5th p. 514 Ed. Note: Ordinarily, you would use particles (e.g., Ivalon) to destroy the nidus of an AVM. The key difference with a pulmonary AVM is that any stray particles (and some will get through) may embolize to the CNS, causing a stroke [recall that stray particles from a peripheral AVM would be caught by the lung].Good quick review: Abnormal communication b/w pulmonary artery and veins. Types: congenital (60%) Osler-Weber_Rendu disease (hereditary hemorrhagic telangiectasia). Acquired (40%): iatrogenic, infection, tumor. Radiograpic pearls: lower lobes 70% > middle > upper. Feeding artery with feeding veins. Sharply defined mass, strong enhancement, change in size with Valsava / Mueller movement. Primer 63-64
• Indications for TIPS include all except:
a. Intractable ascites
b. Bleeding esophageal varices with failed sclerotherapy
c. Gastric varices from splenic vein thrombosis
d. Refractory ascitic hydrothorax
c. Gastric varices from splenic vein thrombosis
• Patient has a TIPS placed from R portal vein to R hepatic vein. What is the flow in the L portal vein?
a. Hepatofugal
b. Hepatopetal
c. Absent
d. Biphasic
• A. Hepatofugal “Hepatofugal flow is frequently seen in the portal vein branches after successful TIPS. Alteration in flow from hepatofugal to hepatopetal may indicate stent malfunction or occlusion.” see Dähnert p. 735.Biphasic flow is seen in arteries with either vasoconstriction or acute occlusion distal to the sampling site. Source: McGahan, Diagnostic Ultrasound p. 683, 998Remember, hepatopetal (from petere, Latin “to seek”) is into the liver (normal)
- hepatofugal (from fugere, Latin “to flee”) is out of the liver (reversed)
• Which of the following is associated with the highest risk when a patient undergoes a pulmonary arteriogram?
a. LBBB
b. RBBB
c. A-fib
d. Pulmonary arterial hypertension > 70 mm Hg
• D. Pulmonary arterial hypertension > 70mm Hg.
• Under normal circumstances the death rate of PA is 0.1-0.5% which increases to 2.3% if severe pulmonary arterial hypertension is present defined as PA systolic pressures > 70 mm Hg (pulmonary HTN = sustained systolic PAP ≥ 25 mm Hg) Dähnert 5th p. 572. If patient has LBBB can prophylactically be transjugularly paced prior to procedure.Source: Kandarpa, Handbook of IR procedures 3rd ed p. 35-42
Coronary Artery Anatomy [pic]
1)Right Coronary aa (RCA) 2)Left main coronary aa (LCA)
3)Circumflex artery (CFX) 4) Left anterior descending aa (LAD
5) Posterior descending aa (PD) 6) Acute marginal aa
• A patient with a left circumaortic renal vein has a lower extremity DVT. The anterior left renal vein is at the same level as the right renal vein. The best location for the tip of the IVC filter is?
a. Between the right renal vein and anterior left renal vein.
b. Between the two left renal veins
c. At the level of the inferior left renal vein
d. Below the inferior left renal vein
e. Above all three veins
f. Two filters, tip of one below the right renal vein and the tip of the second below the retroaortic left renal vein.
• C. At the level of the inferior left renal vein Circumaortic left renal vein is found in ¾% of people with lower component of the venous ring lying behind aorta and draining into the IVC lower than a “normal” renal vein. Place the apex of the filter just at or slightly below orifice of the lowest renal vein.
• Which of the following regarding the IMA is false:
a. Commonly occluded in people over age 50
b. The IMA provides supply to the transverse and descending colon
c. Origin at the level of L3
d. Originates from anterolateral aorta
e. Communicates with internal iliac arteries via collaterals
• B. The IMA provides supply to the transverse and descending colon. (FALSE)This choice is definitely false. The superior mesenteric artery supplies the right colon from the cecum to the splenic flexure. The inferior mesenteric artery supplies the left colon from the splenic flexure to the rectum. B&H 2nd p. 759.The IMA originates at the level of the left pedicle of L3 (Primer 3rd p. 676)Occlusion of the IMA is certainly not unusual—abundant collaterals maintain colonic blood supply. E.g., the IMA is occluded in 80% of AAA (Dähnert 5th p. 605).Collateral path of IMA is to hemorrhoidal to the internal iliac to the external iliac arteries. Source: Kandarpa, Handbook of IR Procedures 3rd ed p.705
• Which of the following is false regarding percutaneous nephrostomy?
a. The anterior and posterior segmental arteries can best be avoided by entering an anterior calyx.
b. The posterior calyces are viewed en face in the AP projection
c. It is the treatment of choice for acute pyonephrosis
d. There is an increased incidence of subsequent infectious complications in patients with bladder dysfunction.
e. Puncture above the 10th rib should be avoided
• A. The anterior and posterior segmental arteries can best be avoided by entering an anterior calyx. The puncture should follow Brodel’s Line (an oblique posterior-lateral approach) through the avascular zone of the kidney. Source: Kandarpa, Handbook of IR Procedures 3rd ed p.280
• “The optimal access should enter a posterior calyx on end…”. This is why the patient is prone on the fluoro/angio table! see Ferral p. 263. Puncture above the 10th rib may enter the pleura (pneumothorax).
• Patient has an angiogram of the right CCA which shows occlusion of the right proximal ICA. The intracranial portion of the right ICA is visualized on the same injection. Why?
a. Filling through the ipsilateral ophthalmic artery
b. Filling through the contralateral ophthalmic artery
c. Filling through the anterior communicating artery
d. Filling through the ipsilateral temporal arrtery
• A. Filling through the ipsilateral ophthalmic artery.
• Collateral paths in proximal ICA occlusion/stenosis include:
• Maxillary artery to pterygoplantine branches to ethmoidal branches of ophthalmic to supraclinoid ICA
• Anterior branch of superficial temporal to Carotid
• MMA to lacrimal branches of ophthalmic
• Source: Osborn, Intro to Cerebral Angio p.85
• Regarding a peripherally inserted venous catheter, which of the following is false:
a. PICC lines are used for intermediate access (2-26 weeks)
b. Tunneled catheters have separate skin and vessel entry sites, thereby lowering infection rates
c. Both central venous and line patency can be prolonged using low-dose coumadin
d. The incidence of line infection is higher with TPN than with chemotherapy
e. The rate of line infection is lower for lines placed in the O.R. than for lines placed in the angio suite.
• E. Infection rate is lower for lines placed in the O.R. than in the angio suite.Source: Kandarpa, Handbook of IR Procedures, 3rd ed p.465-471
• Infectious complications of central line placement in radiology are no higher than the 10-15% rate of catheter-related septicemia reported in surgical series Interventional Radiology – Castañeda (pg 1388).
• Chemoembolization in a patient with hepatic metastases from a neuroendocrine tumor would be most helpful for:
a. Pain related to tumor bulk
b. Hormonal symptoms
c. Failed chemotherapy
d. Evidence of rapid tumor growth
• B. Hormonal symptoms Carcinoid tumor mets to the liver cause Carcinoid syndrome (syndrome can also be induced by primary pulmonary or ovarian carcinoid). Symptoms include diarrhea, right heart failure, bronchospasm, wheezing, nausea and vomiting, hypotension, flushing.Treatments include Somatostatin/SMS 201-995; chemoembolization of hepatic arteries. Source: Dähnert 5th ed. p.802
• The most widely accepted indication for percutaneous, catheter-directed staining and ablation is:
a. A patient s/p neck exploration with persistent hyperparathyroidism and a mediastinal parathyroid adenoma
b. Preoperative embolotherapy
c. Functioning parathyroid adenoma
d. Non-functioning parathyroid adenoma
• A. S/P neck exploration with persistent hyperparathyroidism and a mediastinal parathyroid adenoma.> 90% of patients with parathyroid adenoma cured by first surgery. “Staining” refers to infusing contrast directly into adenoma causing extravasation and ischemic injury. Source: Abrahms Angio, p.976-991
• In angiographic ablation, the feeding artery is superselectively catheterized and high-osmolar ionic contrast material is injected to induce ischemia and infarction of the gland… Generally, radiologic procedures are selected when the patient is a high-risk surgical candidate. http://www.emedicine.com/radio/topic525.htm
• Regarding abdominal aortic grafts, which of the following are false:
a. There is 1-2% incidence of post-op graft infection
b. Graft infection carries a 30-70% mortality rate
c. Staph. species can be related to gas adjacent to the graft
d. Gas adjacent to the graft 3 weeks after surgery is indicative of infection
e. Fluid adjacent to the graft 3 months after surgery indicates infection.
• D. Gas adjacent to the graft at 3 weeks indicates infection
• There can be “normal post-op ectopic gas” for 3-4 weeks (usually disappers by 4-7 weeks). The incidence of graft infection is 1.3-6% with a 17-75% mortality rate. There is complete resolution of hematoma by 3 months.Source: Dähnert 5th ed. p.610
• Which of the following vessels does not directly feed into the popliteal vein?
a. Lesser saphenous
b. Gastrocnemius (Sural) vein
c. Soleal vein
d. Tibioperoneal trunk
• A. Lesser saphenous (or Soleal vein?)The veins of the leg follow the arteries and are named accordingly.Deep veins: Ant tib, Post tib, peroneal and calf (Soleal and Gastrocnemius) All drain into the popliteal. The Soleal vein may drain in to the post tibial + peroneal veins or lower part of the popliteal vein. Superficial veins: Greater/lesser saphenous. The lesser saphenous does drain into the popliteal vein 60% of the time.
Source: Dähnert 5th ed. p.600-602
• Regarding vascular anatomy, which of the following is true?
a. The right iliac artery lies posterior to the iliac vein
b. The right testicular vein lies posterior to the ureter
c. The left renal vein lies posterior to the aorta
d. The SVC lies anterior to the right pulmonary artery
• D. The SVC is anterior to the right PA
• Source: Netter plate 201; Weir, Imaging Atlas of Human Anatomy 2nd ed. p.94, frame G
• A 24 y/o military recruit has pain in the calf after running 2 miles. The most likely etiology is:
a. Buerger’s disease
b. Effort thrombosis
c. Raynaud’s phenomenon
d. Popliteal artery entrapment syndrome
• D. Popliteal artery entrapment syndrome
• Classic presentation is claudication in calf and foot of a young man (9M:1F) with onset of symptoms sudden and occurring during episode of peak lower extremity activity. Bilateral in up to 66%. Dähnert 5th p. 643 Resnick p. 138.
• Effort thrombosis (aka Paget-Schroetter’s disease) is thrombosis of axillary/SCV usually in males due to progressive damage by repetitive exercise which compresses vessel against scalene or subclavian tendon.
Source: Rutherford Vascular Surgery p.999-1001
• The vagus nerves passes through the diaphragm along with what structure?
a. Aorta
b. Esophagus
c. Azygous
d. Thoracic duct
e. Hemiazygous
• B. Esophagus Aorta and thoracic duct pass together.Azygous and Hemiazygous veins individually perforate diaphragm.Source: Netter, plate 228
• A patient with a DNR order arrives in IR for a procedure. What is the most appropriate thing for the radiologist to do?
a. Cancel the DNR order
b. Discuss with the patient and family whether or not the DNR order should remain in effect during the procedure
c. Refuse to do the procedure
• B. Discuss with the patient and family.
No reference, just common sense.
• Regarding FMD of the carotid arteries. Which of the following is false?
a. String of beads appearance
b. ICA more common than ECA
c. Unilateral
d. Intracranial saccular aneurysm
• C. Unilateral. (False)
• The most common CNS vessel affected is the internal carotid artery, approximately 2 cm from the bifurcation in 90% of cases, with rarer involvement of the external or common carotid arteries, and vertebral artery involvement in 12% of cases, and bilateral carotid artery involvement in 60% of cases (Requisites p116)
• Fibromuscular displasia produces an appearance characterized as a “string of beads,” most often with hyperplasia of the media, although thinning of the media with associated disruption of the internal elastic lamina produces saccular dilatations, or pseudoaneurysms (Requisites p116).
• Multiple T/F—With a carotid-cavernous fistula on angiogram:
1. Reversal of flow in the ophthalmic vein.
2. Early filling of the sagittal sinus.
3. Filling of the contralateral cavernous sinus.
4. Contralateral MCA injection will have cross-filling of the ipsilateral ACA across the ACOM.
5. Early draining at the vein of Galen.
• A carotid-cavernous sinus fistula represents abnormal communication between the veins of the cavernous sinus and the internal carotid artery. Dähnet 5th p. 338
• .1. True. There will be retrograde filling in the ophthalmic vein and any other veins that drain into the cavernous sinus, including the opposite cavernous sinus through the intercavernous sinus.
• 2., 5. False. The superior saggital sinus nor the vein of Galen should not fill early as they do not communicate with the cavernous sinus until the level of the transverse sinus
• 3. True. Basically, everything that communicates with the cavernous sinus involved will attempt to decompress the fistulous blood. The cavernous sinus are connected by the intercavernous sinus, among other routes. Clemente 4th plate 488, 490.
• 4. True. Contralateral ICA injection is useful in determining extent of collateral flow. (Requisites p. 63 and 295-6).
• Which is not associated with unilateral rib notching?
a. Blalock-Taussing shunt
b. Peripheral AVM
c. Brachial artery stenosis
d. Coarctation with aberrant right subclavian artery
e. Coarctation between the innominate and left carotid.
• C. brachial artery stenosis
• Collateral vessels from the aortic branches proximal to the obstruction enlarge in an attempt to provide adequate flow to the aorta distal to the coarctation. The intercostals arteries are a component of this collateral network and cause of the notching and sclerosis of the undersurface of the third through eight ribs posteriorly. Pg 19 Danhert: 5th ed
- since the brachial artery commences at the lower margin of the tendon of the teres major (beyond the take-offs for the intercostals), a stenosis in that area would not cause collateralization through the intercostals. Arterial source causes of rib notching include the aorta (coarctation, thrombus of the aorta), subclavian artery ( Blalock Taussig shunt ) and pulmonary artery (pulmonary stenosis, TOF, absent pulmonary artery). Venous sources of rib notching include an AV malformation of the chest which leads to enlargement of the intercostals veins.
• Vasospasm is induced by a guidewire during angioplasty of the popliteal artery. How would you treat this?
a. nitroglycerin (NTG)
b. papavarine
c. lidocaine
d. ASA
e. Tolazoline
f. Epinephrine
g. pentolamine
• A. nitroglycerin (NTG)
• NTG is direct smooth muscle relaxant and is more effective vasodilator than tolazoline and is safer with a short half life (dose 200ug in 10 ml saline)
• Drugs for Vasospasm:
Direct acting: ntg, papavarine, nitroprusside, hydralazine, isoproteronol
Sympatholytic drugs: tolazoline (priscoline), phentoloamine, reserpine, phenoxybenzamine
Calcium channel blockers: verapamil, nifedipine, Tolazoline is a alpha adrenergic blocker and is a patent vasodilator with a duration of several minutes after intraarterial injection (dose 25-50mg in 10ml saline). Heart disease is contraindication. Has potential for inducing hypotension, tachycardia, arrythmias and MI. Has several hour half life. Papavarine is also direct smooth muscle relaxant with half life between 30 min and 2 hour. ASA inhibits platelet aggregation. Periarterial injection of lidocaine can reduce local spasm. Epinephrine - sympathomimetic drug with both alpha (vasoconstriction) and beta (vasodilation) adrenergic receptor responses. Vasoconstriction is most pronounced in cutaneous, renal & visceral vessels. Upon administration of 3-12 of epi, the small and mid-sized vessels constrict. Ref: Kadir 370-372; Wojtowycz (Woj) 33-34
• Nonocclusive mesenteric ischemia (NOMI) is manifest by all except:
a. nonstenotic irregularity at the vessel origin
b. irregularity/defects in transmural bowel vessels
c. decreased filling and irregularity at intestinal branch points
d. nonfilling of mesenteric vessels following papavarine
d. nonfilling of mesenteric vessels following papavarine
• Axillary/subclavian effort thrombosis is not associated with:
a. physical activity
b. more common in older women
c. may be associated with vessel compression at thoracic inlet
d. can be initially treated with thrombolytics
e. increased with stress
f. is not related to central venous catheter insertion
• B. more common in older women (False)Many names for this disorder: effort thrombosis of subclavian vein, Paget Schroetter syndrome, primary axillary-subclavian vein thrombosis, gouty phlebitis, spontaneous subclavian vein thrombosis
- Typically effects young men (< 40), usually in dominant arm (b false)
- often history of vigorous or unusual exercise (commonly weight lifting) (a true)
- cyanosis of extremity (may only be during exertion), swelling, prominent venous collaterals about the shoulder and anterior chest
- 2 angiographic patterns 1) long segment obstruction of the axillary and subclavian veins 2) short segment obstruction of the axillary/subclavian venous segment at the level of the first rib-clavicle junction
- may need to hyperabduct arm (to 180 degree) to produce occlusion at venography (pressure measurements should increase by 10mmHg or more after abduction to indicate disease)
- treat with catheter directed thrombolysis (d true). After treat often find irregularity or stenosis in the subclavian vein at the first rib/clavicle junction (thoracic inlet syndrome). Contributors to development include subclavian and axillary vein compression by costoclavicular ligament, subclavius muscle, and clavicle against first rib, stasis, recurrent trauma, and venospasm Castaneda (Cas) 793-5; Woj 155-6
• What is an indication for TIPS?
a. definitive treatment of cirrhosis and portal HTN
b. treatment of ascites prior to medical intervention
c. bleeding esophageal varices refractory to sclerotherapy
d. prevention of variceal bleeding
• C. bleeding esophageal varices refractory to sclerotherapyIndications for transjugular intrahepatic portal-systemic shunt (TIPS):
- Continuing or recurrent variceal bleeding refractory to medical or endoscopic control
- Massive ascites not responding to medical therapy
- Hepatic veno-occlusive disease; hepatorenal syndrome Dähnert 5th p. 735.A is false because TIPS is not a definitive treatmentB is false since you only use TIPS as a treatment of ascites if it is refractory to mecical interventionD is false since TIPS is unproven in prevention of variceal bleeding Cas 262-5: Woj 103-4
• What is a contraindication to TIPS?
a. Budd Chiari
b. Portal thrombosis
c. Right heart failure
d. INR > 5
• B. right heart failure (best answer)ABSOLUTE contraindications to TIPS
- Rt. Sided heart failure with elevated central venous pressure B&H p.635.
- polycystic liver disease
- large carcinoma infiltrating porta hepatic
- severe hepatic failure with extremely poor hepatic functional reserve
• RELATIVE contraindications to TIPS
- portal vein thrombosis, active intrahepatic or systemic infection, Severe hepatic encephalopathy poorly controlled by medical therapy, stenosis of celiac axis, hypervascular liver tumor. Cas 262-5; Woj 103-4
• Patient has a pelvic fracture with a pelvic hematoma. There is extravasation from a branch of the hypogastric a. What would you do?
a. keep transfusing, the bleed will eventually tamponade
b. coil at the origin of the internal iliac a.
c. embolize with gelfoam or small particles distally in the internal iliac
d. place patient in compression suit in order to tamponade
e. sugically treat
• C. embolize with gelfoam or small particles distally in the internal iliac.
• For traumatic hemorrhage, aim of embolization is not to cause tissue infarction, but to diminish perfusion to the point that endogenous hemostatic mechanisms can stop bleeding. Always deploy agent as selectively and close to bleeding site as possible
• With pelvic trauma use gelfoam or Ivalon particles to accomplish distal embolization (c is true). Extensive collaterals are present in pelvis, therefore a proximal occlusion of the hypogastric a. can still allow persistent extravasation at site of vascular injury distal to the proximal occlusion (b is false) There is nothing in the pelvis to tamponade an arterial bleed-pt can bleed out (a not great answer); a MAST trousers are used to shunt blood from the lower extremities to sustain central perfusion – their purpose is not to tamponade (D not true) Often difficult to find all of bleeding sites surgically and if ligated can commonly have hemmorhage continue through collaterals (e not great) Woj 231-2; Cas 70-71
• Which is the most accurate regarding renal artery angioplasty for treatment of renal vascular hypertension?
a. ostial lesions have a higher success rate than nonostial lesions
b. renal artery calcification is a contraindication
c. angioplasty in FMD has a 30% rupture rate
d. severe hypertension is a recognized complication
e. should not be performed if elevated renin levels cannot be localized
• D. severe HYPERtension is a recognized complication
• Blood pressure should be closely monitored in first 24-48 hr after angioplasty, b/c profound changes can occur. Transient early HTN can be seen in 30% of pts within two hr of procedure and can persist for 24hr (presumably due to renin washout phenomena)
• Lateralization of renin values in selectively obtained samples helps predict a beneficial result, but absence of lateralization should not be seen as contraindication b/c 50% of such pts. Improve after PTA (E is false)
• Ostial angioplasty has poor technical and clinical results b/c it is aortic plaque rather then renal plaque which is subjected to longitudinal displacement from balloon. Considered ostial if in first 5-10mm of renal a. (A is false)
• 5-10% complication rate of renal angioplasty: transient renal insufficiency from contrast, subintimal dissection, thrombosis of r.a., rarely rupture of r.a. (c is false), distal embolization – rare, spasm – common, groin complications B must be false since atherosclerotic dz is the most common etiology of stenosis Woj 194-99; Cast 483-94
• Regarding peripheral AVF, what is false?
a. they may occlude spontaneously
b. loss of triphasic waveform in artery
c. increased flow in artery distal to fistula
d. arterialized venous flow in center of AVF
e. should be treated with coils
f. low resistance waveform in feeding artery
g. increased venous pulsatility
• C. increased flow in artery distal to fistula (False)C is false. There is decreased flow in the artery distal to the fistula.A is true. AVF may occlude secondary to stenosis, with compression, after dialysis, hypovolemia, infection, or hypercoagulabitlity B is true. Normal findings in AVF include monophasic flow with peak systolic velocities 100-400 cm/sec in the supplying a. and throughout the graft.D is true. The draining veins show arterial pulsations with peak velocities of 30-100 cm/secE can be true—permanent occlusion would be a goal of therapy.
• Patient being treated with heparin for PE. Her thigh is tender, diffusely swollen, and there is decreased distal pulses. Patient is 80 – you should:
a. increase heparin
b. stop heparin
c. venogram
d. arteriogram
e. IVC filter
• B. stop heparin“…The most feared complication [of HIT] is thrombosis, either venous or arterial, which occurs in about 20% of the recognized HIT cases. In some patients with HIT there is a dramatic and occasionally immediate drop in the platelet count, often occurring within 6 weeks of previous heparin exposure. This presentation is particularly associated with thrombosis, often the type referred to as the "white clot syndrome", due to a fibrin-platelet rich clot in the arterial circulation.” from http://www.itxm.org/TMU1997/tmu1-97.htmAster RH. Heparin-induced thrombocytopenia and thrombosis [editorial]. N Engl J Med. 1995;332:1374-1376
• Which is a contraindication to TPA:
a. pregnancy
b. CVA in past 2 months
c. INR=1.4
d.
• B. CVA in past 2 months
• From Kandarpa (IR Handbook), 2nd edition (p. 59.):
Absolute contraindications:
#1. Active internal bleeding
#2. Irreversible limb ischemia
#3. Recent stroke within 2 months (some prefer 12 months)
#4. Intracranial neoplasm / recent craniotomy
#5. Protruding mobile left heart thrombus.
Relative contraindications:
#1. GI bleeding (by Hx)
#2. major surgery (within 10 days)
#3. recent trauma
#4. recent CPR
#5. severe uncontrolled high blood pressure
#6. emboli from a cardiac source
#7. subacute bacterial endocarditis
#8. coagulopathy
#9. pregnancy and postpartum period (< 10 days)
#10. severe cerebrovascular disease
#11. diabetic hemorrhagic retinopathy.
• Regarding a peripherally inserted venous catheter, which of the following is false:
a. PICC lines are used for intermediate access (2-26 weeks)
b. Tunneled catheters have separate skin and vessel entry sites, thereby lowering infection rates
c. Both central venous and line patency can be prolonged using low-dose coumadin
d. The incidence of line infection is higher with TPN than with chemotherapy
e. The rate of line infection is lower for lines placed in the O.R. than for lines placed in the angio suite.
• E. Infection rate is lower for lines placed in the O.R. than in the angio suite.Source: Kandarpa, Handbook of IR Procedures, 3rd ed p.465-471
• Infectious complications of central line placement in radiology are no higher than the 10-15% rate of catheter-related septicemia reported in surgical series Interventional Radiology – Castañeda (pg 1388).
• Most Common cause of dialysis graft failure:
a. improper placement
b. venous failure
c. mid-graft thrombosis
d. arterial thrombosis
e. abnormal stick
• B. venous failure Most common location for graft failure is at the venous anastomosis. P. 727 AbramsStenosis of a graft most commonly occurs at the venous anastomosis (in 80%). Failure of a native AV fistula is more likely to be due to stenosis either centrally or at the arterial anastomosis. Castañeda synopsis p. 120
• In type A aortic dissection, MR is more useful than angiography for which of the following? (Multiple T/F)
1. Evaluation of false lumen thrombus
2. Pericardial tamponade
3. Relation of dissection to the coronary arteries
4. Point of origin of dissection
5. Aortic insufficiency
• In type A aortic dissection, MR is more useful than angiography for which of the following? (Multiple T/F)
1. Evaluation of false lumen thrombus
2. Pericardial tamponade
3. Relation of dissection to the coronary arteries
4. Point of origin of dissection
5. Aortic insufficiency

• 1. True. Evaluation of false lumen thrombus – MR can better demonstrate this. Further, a completely thrombosed false lumen (10%) can give a false-negative on angio.
• 2. True. Pericardial tamponade – Neither MR nor angio is great, but MR is better because of fluid on T2.
• 3. False. Relation of dissection to the coronary arteries – Angio would better demonstrate this relationship. MRI is limited with small vessels. Dähnert 5th p. 609.
• 4. False. Angiography is superior to any other technique in demonstrating entry and reentry points.
• 5. False. Aortic insufficiency - Even with the cine capability of MR, they felt angio was still better with real time contrast injection. see Card. Req. p. 392-398.The diagnosis can usually be made using conventional spin-echo MR technique with electrocardiographic gating and T1-weighting; however, slow flow of blood within the false lumen may mimic intramural thrombus on spin-echo MR imaging. The distinction can readily be made using gradient-echo MR imaging, cine phase contrast imaging, or gadolinium-enhanced MR angiography. [Fraser and Pare, p. 2957]
• False aneurysms are concerning because:
• A. tend to rupture. Chronic posttraumatic aortic pseudoaneurysm = aneurysm existing for > 3 months. 2-5% of patients surviving aortic tear > 24-48 hrs. Symptom free period of months to years. Delayed clinical symptoms. Usually at level of ligamentum arteriosum filling AP window. May develop CHF, partial obstruction of aortic lumen, bacterial endocarditis, aortoesophageal fistula, aortic dissection, obstruction of tracheobronchial tree, systemic emboli. Prognosis = enlargement and eventual rupture. 10 yr survival is 85% with surgical repair and 66% without repair. Dähnert CD ROM
• Regarding blood born pathogens, which is false?
a. HIV is more infectious than Hep B
b. Injury with a hollow needle is worse than with a scalpel
c. General surgeons have a higher rate of exposure than Interventional Radiologists
d. Without blood, urine and stool are not infectious
e. Blood, semen, amniotic fluid and CSF are contagious
• A. False
• B. True. Risk of infection is directly related to pathogen “load,” which is higher with a hollow needle.
• C. True.
• D. False. True see slide # 37
• E. True.
• A patient with a Mallory-Weiss tear and UGI bleeding is sent for angiogram. What is the most likely vessel bleeding?
a. Hepatic artery
b. Adrenal artery
c. Inferior phrenic artery
d. Left gastric artery
e. Inferior esophageal artery
f. Gastroduodenal artery
g. Coronary vein
h. Azygous vein
• D. Left gastric artery
• With a Mallory-Weiss tear, a linear collection of extravasated contrast at the GE junction on left gastric arteriography. The inferior phrenic artery is a branch of the left gastric artery. (LaBerge 306).
• Which is the most potent Vasoactive Substance?
• Epinephrine
• Norepinephrine
• Angio 1
• Angio2
• Renin
• D. Angiotensin II Epinephrine causes vasoconstriction and increased cardiac output as an alpha and beta adrenergic agent. It is useful for adverse contrast reactions including severe urticaria, respiratory distress and anaphylaxis. Norepinephrive has similar actions but is not administered IV or usually even discussed. Renin mediates conversion of angiotensin to angiotensin-I. A reninoma causes HTN, hypernatremia and hypokalemia via secondary hyperaldosteronism.Angiotensin-I is converted to Angiotensin-II by an ACE. Angiotensin-II is the most potent vasoconstrictor of biologic system. Dähnert p 752, back cover, Weissleder p267
• Which of the following is true?
• Embolization with small particles to avoid tissue necrosis
• Pulmonary AVM’s should be embolized with gelfoam
• Small bowel embolization should be performed with coils
• Treatment of varicococeles should be with permanent embolization
• D. Treatment of varicococeles should be with permanent embolization. Embolization with small particles can result in occlusion at the capillary level and presents a definite risk for ischemia and infarction. Pulmonary AVNs are embolized with detachable balloons or coils, other embolic materials would pass through the malformation. Mechanical embolization techniques are used in GI bleed for failure of pharmacologic treatment. Gelfoam plegets or microcoils can be used in such instance. For most splanchnic embolizations, Gelfoam is the appropriate embolic agent. Gelfoam soaked in a broad spectrum antibiotic is the agent of choice for mesenteric embolization.Varicoceles require permanent embolization. Commonly use materials include sclerosing agents, coils, ballons, hot contrast and PVA sponges.Kadir, S. Teaching Atlas of Intervensional Radiology. 1999 pg 307.Agents for Small Vessel and Tissue Embolization and Mechanical Embolic Agents. General considerations. Volume III. Abrams’ Angiography. 1998.
• Treatment of pulmonary AVM includes:
a. Coils
b. Gelfoam
c. Ivalon particles
d. Ethyl alcohol
• A. Coils This question has been remembered by most to include the term Pulmonary AVM. The other treatment option for pulmonary AVM is detachable balloons (permanent occlusion). Be aware that there are a multitude of AVM locations with each locale classically using different embolic material-i.e. intracranial AVM you most commonly use glue or particles or extracranial AVM you most commonly use particles. In some, alcohol / PVA is used to destroy the nidus (only blocking inflow allows collaterals to develop).Source: Dähnert 5th ed. p.514, Kandarpa: Guidebook of IR procedures, 3rd ed. p120-122.
• During an arteriogram, the patient becomes tachycardic and hypotensive Do all the following except:
• Trendelenberg
• Oxygen
• Epinephrine
• Atropine
• IV fluids
• D. Atropine For an anaphylactoid reaction (tachycardia, hypotension, dizziness) give fluids, oxygen, and epinephrine SQ. Don’t give Atropine, it is used for vasovagal reactions (bradycardia). Dähnert back cover.
• Regarding embolization:
• Use particles as small as possible to decrease the risk of tissue necrosis
• Pulmonary AVM success is the same as peripheral AVMs
• There is a risk of stroke
• C. There is a risk of stroke
• a. False: General Principles: proximal occlusion is equivalent to surgical ligation – it does not compromise collateral flow. Distal embolization usually causes tissue necrosis and infarction. Be as selective as possible. Smaller particles increase risk of necrosis Brant and Helms pp 623-625.
• b. False: On multiple prior tests, pulmonary AVMs do not have the same recurrence rate as peripheral AVMs. I however, can’t find this stated in Gamsu, Dähnert , Brant and Helms or Weissleder.
• c. True: As with any embolization there is always a risk of stroke
• In a patient, you are doing local freezing and suddenly the patient becomes dizzy with a bp of 70/40 and bradycardic. Which of the following is NOT included in the most appropriate treatment.
• oxygen
• IV fluids
• Atropine
• Epinephrine
• D. Epinephrine
• For vasovagal reactions (bradycardia, hypotension and dizziness) IV fluids, atropine and the trendelenberg position are appropriate. Epinephrine is not mentioned. Dähnert back cover
• Most common cause of dialysis graft failure:
• improper placement
• venous outflow failure
• mid-graft thrombosis
• arterial thrombosis
• abnormal stick
• B. venous outflow failure
• Grafts have a finite life span with an average patency of 20 months. A stenosis at the venous anastamosis is the most common underlying lesion. The next most frequent sites of stenosis are the outflow veins, followed by the central veins. Brant/Helms pp 630 –631.
• Peripheral venous access all are true except :
• Increased infection rates when done in IR vs. surgery
• tunneling the catheter will decrease infection rates
• coumadin can decrease the incidence of thrombus formation around the catheter tip
• PICC is for intermediate (2-51 week) access
• increased infection rates with TPN vs chemoTX
• Peripheral venous access all are true except :
• Increased infection rates when done in IR vs. surgery
• tunneling the catheter will decrease infection rates
• coumadin can decrease the incidence of thrombus formation around the catheter tip
• PICC is for intermediate (2-51 week) access
• increased infection rates with TPN vs chemoTX


• a. False: per Abrams angiography
• b. True
• c. True: Coumadin can decrease the chance of central venous catheter thrombosis but you don’t NEED to give it.
• d. True: A PICC is for short term use but can be left in longer than 20 weeks.
• e. True:. This has been the consensus answer for this choice in many year’s questions, without any references. A quick InterNet search suggests the following reasons TPN has higher infection rates: #1 co-morbidity, #2 line sepsis, #3 infectious agents in the TPN solution, and #4 hyperglycemia from the TPN.Brant/Helms pp 633-635.
• The superior hemmorhoidal art is a branch of what?
• celiac
• SMA
• IMA
• internal iliac
• The superior hemmorhoidal art is a branch of what?
• celiac
• SMA
• IMA
• internal iliac


• C. IMA
• Weissleder p 624
• Regarding traumatic rupture of the aorta
• patients who are not treated will die
• A mediastinal hematoma means there is certain aortic injury
• A. Non treated will die
• a. True: In a complete rupture, 85% of patients exsanguinate before they reach the hospital.
• b. False: The source of mediastinal hematoma is usually venous. Aortic injury is the cause in only 12%. . Dähnert p 513
• Factors included in the selection of an IVC filter include all of the following except:
• Location of IVC thrombus
• Variant venous anatomy
• Position of the renal arteries
• Size of thrombus
• Size of the IVC
• D. Size of thrombus
• A. TRUE. If a clot is found in the IVC, the filter must be placed above the clot through the internal juglualr vein which would affect filter choice
• B. TRUE. Variants of IVC should be sought. Duplicated IVC’s require placement of filters in each vessel.
• C. TRUE: Position of the renal arteries has to be considered.The staging section for IVC filter placement in Valji does not mention renal artery position. However, location and number of renal veins is important. Filters are deployed with the most superior point immediately below the lever of the renal veins. If the filter clots spontaneously or becomes filled with emboli, rapid flow from the renal veins should prevent thrombosis above the filter if no dead space is left (Valji ’99, pg. 304-6).
• D. FALSE: Chronic total occlusion of the IVC obviates the need for filter placement BUT clots can pass through enlarged collaterals, So I guess in this sense size\volume of clot should be considered. ALSO FALSE per many old test
• E. TRUE. About 1-3% have a megacava. In such cases, the Bird’s nest filter should be used. Alternatively, smaller filters can be placed in each common iliac vein. The Bird nest filter system is longer than other devices and may not be suitable if the infrarenal IVC is short. So in length and width, the size of the IVC can be important.
• Patient presents with Varicocele, what is the treatment
• Permanent occlusion of spermatic vein
• Orchiectomy
• Alcohol ablation
• Temporary occlusion of spermatic vein with gelfoam
• A. Permanent Occlusion Of Spermatic Vein
• A person has obstructed stone with pyonephrosis. After the administration of antibiotics, the internist requests a precutaneous nephrostomy. Your response would be to:
• state that you should do it after the system has been on Abic for a few days
• attempt to extract the stone while you are there
• do it but warn the physician of the possibility of progressive sepsis
• C. do it but warn of progressive sepsis
• Which of the following is false regarding percutaneous nephrostomy?
a. The anterior and posterior segmental arteries can best be avoided by entering an anterior calyx.
b. The posterior calyces are viewed en face in the AP projection
c. It is the treatment of choice for acute pyonephrosis
d. There is an increased incidence of subsequent infectious complications in patients with bladder dysfunction.
e. Puncture above the 10th rib should be avoided
• A. The anterior and posterior segmental arteries can best be avoided by entering an anterior calyx.
• Where should the tip of the aortic balloon pump be located:
• at the root
• at the arch
• just distal to knob
• at the diaphram
• c. just distal to the knob
• Of the following, which pose the greatest risk to the patient about to undergo pulmonary arteriography?
• Pulmonary arterial systolic pressure greater than 70 mmHg.
• Left bundle branch block.
• Right bundle branch block.
• A Pulmonary arterial hypertension > 70mm Hg.
• Concerning renal transplants, which of the following does not cause reversed renal artery flow:
• acute rejection
• renal vein thrombosis
• renal arterial stenosis
• renal arterial stenosis
• The least likely to have a favorable result from angioplasty (or, the least amenable to angioplasty):
• A short segment (less than 1 cm)
• Calcified proximal lower extremity stenosis.
• A 7 cm common iliac stenosis.
• A short segment stenosis in a patient with an elevated C-reactive protein.
• C. 7cm iliac stenosis A lesion of this length is typically treated with primary stenting. Note that the answer would be the same for a similar lesion in another (e.g., femoral) artery Kadir, Teaching Atlas, pg. 381; 1999In Kadir, page 397 there is a table listing categories of lesions with respect to PTA results in the femoral and popliteal arteries. Basically this is a distal, long segment lesion which has a high early failure rate and is typically a surgical lesion unless the patient has contraindications. Kadir, Teaching Atlas, 1999
• Systolic/diastolic pressures in the main pulmonary artery.
• 20/0 mmHg
• 20/8 mmHg
• 60/20 mmHg
• B. 20 / 8 mmHg Best answer is b as the reported numbers are as follows: systolic 20 mmHg, diastolic 10 mmHg, Mean 14 mmHg, capillary wedge pressure 5mmHg. Incidentally, PAH is a systolic pressure >30mmHg and on the CXR the main artery is >29mm. Weissleder, page 60
• Concerning aortic dissection, which of the following vessels is least likely to be involved?
a. Left coronary artery
b. Left renal artery
c. Left common iliac artery
d. Left subclavian artery
e. Brachiocephalic artery
• A. Left Coronary Artery
• Helical flow pattern of blood starting anteriorly proceeding to the right lateral wall of the aorta to the superior and posterior walls of the arch and terminating in the the left iliac artery 80% of the time. 50% of all cases involve the left renal artery.
Source: Dähnert 5th ed. p.608
• Which of the following is false regarding venous thoracic outlet syndrome?
a. thrombolysis is an accepted treatment
b. results from occlusion of a vein
c. commonly occurs with exercise
d. most commonly seen in elderly females
• D. Most commonly seen in elderly females.
• Kawasaki’s syndrome is associated with EXCEPT
• gallbladder hydrops
• myocarditis
• coronary artery aneurysm
• renal failure
• pleural effusions
• renal failure
• Pt with splenic v thrombosis and bleeding gastric varices. You should do:
• Splenorenal shunt
• Mesocaval shunt
• Portocaval shunt
• Splenocaval Shunt
• Splenectomy
• Splenectomy
• Indications for TIPS include all except:
a. Intractable ascites
b. Bleeding esophageal varices with failed sclerotherapy
c. Gastric varices from splenic vein thrombosis
d. Refractory ascitic hydrothorax
c. Gastric varices from splenic vein thrombosis
• What happens when a patient has feet claudication (which of the following are true?)
• Independent risk factor for coronary artery disease
• Smokers do better than diabetics with treatment
• Smokers do worse than diabetics with treatment
• A. Independent risk factor for CAD.
• A patient is status post angiogram for coarctation of the aorta and presents with pneumotosis and portal venous air. The next step in management is
• Ultrasound of the arteries
• Cross table lateral to assess for bowel perforation
• B. cross table lateral
• These signs and symptoms are suggestive of ischemic bowel. A cross-table lateral is a reasonable study to order. Ultrasound of the mesenteric blood supply has limited usefulness
• Regarding percutaneous angioplasty:
• Focal intimal disruption is an adverse effect
• Stenting of popliteal and tibial arteries is effective to treat restenosis
• Pain at the site of angioplasty beginning at the time of angioplasty and persisting one hour afterward may not be pathologic
• C. Pain may not be pathologic
• A. False: listed as a mechanism by which balloon angioplasty enlarges the arterial lumen
• B. False: Studies show no greater than 50% success
• C. True: Couldn’t find a reference on this one but it is the only one that’s leftInterventional Radiology Essentials, Laberge, 2000 p148,179
• Which is not true regarding pulmonary arteriovenous malformations?
• Associated with peripheral telangiectasia
• May be hereditary
• Hemoptysis
• May recur
• Use coils for embolization
• May recur
• Not an indication for TIPS
• refractory ascites
• persistant right effusion from ascites
• refractory bleeding varices
• splenic v thrombosis from chronic pancreatitis with varices
• D. splenic vein thrombosis
• Regarding transrectal and transvaginal abscess drainage, all true except:
• Should not give antibiotics until get a sample
• Relatively contraindicated in Crohn’s
• Should be a self-anchoring catheter
• Relatively contraindicated in coagulopathy
• Sedation should be used for pain control and anxiety
• A. Should not give ABX until you get sample
• Regarding non-occlusive mesenteric ischemia all are true except:
• Multiple stenoses near the origin of the SMA
• Non-filling of mesenteric arcades
• Poor filling of mucosal vessels
• No change in findings after long Paparverine treatment
• No change in findings after long Paparverine treatment
• Which of the following is an absolute contraindication to catheter thrombolysis:
• Heme-positive stool within last 12 months
• Pregnancy
• Brain infarct within the last 2 months
• C. Brain infarct in last 2 months
• This is listed in the absolute contrainidcations. (Radiology, ed. Tavaras (2) 146:2)
• Most likely cause of death in a patient with a pelvic ring fracture:
• Hemorrhage
• Pulmonary hemorrhage
• Sepsis
• Fat embolism
• A. Hemorrhage
• With unstable pelvic injuries, especially straddle, malgaigne (or other anterior and posterior ring injuries), and sprung pelvis. (ACR 25, pp. 118-9).
• Regarding percutaneous transhepatic biliary drainage (PTBD):
• Preferred location for puncture is centrally
• With multiple strictures, passing catheters using bilateral punctures may lower serum bilirubin.
• For a lesion obstructing the left ductal system, left subxyphoid and right subcostal approaches should be used.
• If pus is present in the biliary ducts, attempt to bypass the obstructing lesion should be made immediately.
• C. True. “right intercostal and left sub-xiphoid approaches should be used”. Old Questions.
• A. False: “the duct should be punctured in a peripheral location both to gain a sufficiently long length of duct above any obstructing lesion and also to avoid injury to the large portal venous and arterial structures, which are concentrated in the hilum of the liver” Inverventional Radiology Essentials Laberge 2000 p340
• Regarding sharps injuries and recapping, the following are true except:
• Should use the one handed method for recapping
• Usually stick your dominant hand
• Sharps should be placed on a special area on the table when doing procedures
• Should have a special container for disposal
• B. usually stick dominant hand (False)No reference just common sense BUT I think the most current recommendation is not to recap at all.
• Most likely cause of dialysis graft dysfunction
• a. PSA
• b. Stenosis at the arterial anastomosis
• c. Stenosis at the venous anastomosis
• d. Stenosis more proximally in the central draining vein
• e. mid graft thrombosis
Stenosis at the venous anastomosis
• Pt. is status post right transaxillary interventional procedure and presents soon after with right hand weakness and numbness and decreased pinprick sensation. This is most likely from:
• a. Hemorrhage
• b. Nerve damage
• c. Thrombosis
• d. Pseudoaneurysm
• A. Hemorrhage. Axillary artery sticks have a much higher complication rate than do femoral sticks as the artery is smaller and more mobile. hematoma causes compression of the nearby brachial plexus is the most likely and 2nd most common complication of axillary artery stick (68%).
• Nerve damage – false, although nerve damage can be caused by direct needle stick or by hematoma, the answer is hematoma. as it is most likely.
• Pseudoaneuysm – false, this is a much less common complication (22%)
• Thrombosis – false, although this is the most common complication of axillary artery stick (76%) arterial obstruction due to embolization, the radial pulse would be diminished or absent.Primer, 3rd, 643 and Abrams p 1047.
• Regarding a normal angiogram of the popliteal artery to detect popliteal entrapment:
• repeat angiogram in active plantar flexion
• repeat angiogram in active inversion of ankle
• repeat angiogram with leg elevated
• A. repeat angiogram in active plantar flexion. Popliteal entrapment syndrome results from an abnormal relationship between the popliteal artery and the gastrocnemius (or less often, the popliteus) muscle, causing arterial compression. Entrapment is bilateral in 25% of cases, and men are affected more frequently than women. Classic findings are medial deviation of the midportion of the popliteal artery and stenosis; the popliteal vein is usually not affected. Unfortunately, initial angiogram may be normal. A combination of passive dorsiflexion of the foot with active plantar flexion against resistance elicits marked narrowing or occlusion. Valji K,
• Which of the following will have the best outcome with PTA:
• 5 cm stenosis of the iliac artery in a 50 yo man
• Diabetic with long segment stenosis
• Multifocal disease of the tibial artery
• Renal ostial lesion
• Focal stenosis of the iliac artery in a 50 yo with claudication
• E. Focal stenosis of the iliac artery in a 50 yo with claudication.The patient that makes the best candidate for PTA are ones with focal short segment stenosis and claudication . Answer (a) is incorrect because 5cm is considered too long to be ideal for PTA (see above). Answer (b) is incorrect because although diabetes is not a contraindication to PTA, it does not make the patient a better candidate. Also, a long segment stenosis is less amendable to TPA then short segments. Aswer (d) is wrong because single lesions are more amendable to TPA then multiple. Answer (d) is incorrect because lesions at the origin of the renal artery have a much less success rate than other locations of the renal artery. Primer, 3rd, 651-652 and Castenda-Zuniga, 2nd, 373, 379
• Concerning angioplasty of renal artery (T/F):
• Helpful in cases of FMD
• Not used in cases where renin sampling of renal veins does not localize
• Can lead to acute hypotension
• 1=T 2=F 3=T According to Dunnick, “patients with FMD almost uniformly have good results from PTA (percutaneous transluminal angioplasty).” Lesions are easier to approach than in the cases of atherosclerotic disease because of younger patient age, and thus less tortuosity and plaques. 25-35% cured, ~ 60% improved. Renal vein renin measurement tests can miss the detection of renal artery stenosis (false negative) in up to 50% of cases; so, obviously, just a negative test on this should not preclude the use of PTA in anatomically demonstrated stenoses.
• A rare but catastrophic complication of PTA is the rupture of artery, causing acute hypotension due to massive blood loss and needs emergent open surgery. Before renal angioplasty , the patient’s blood pressure should be controlled by angiotensin-converting enzyme inhibitors, such as captopril or enalapril, whenever possible. These agents inhibit the production of angiotensin II, the active hypertensive substance in renin-mediated renovascular hypertension. The pharmacologic action of these agents is physiologically the same as that of a successful renal angioplasty. For this reason, sudden drastic drops of blood pressure due to removal of the renin-angiotensi-aldosterone effect following successful angioplasty are not as pronounced when patients are treated with these drugs.Primer, 3rd, 652 and Dunnick, Textbook of Uroradiology, 2nd ed., p. 215-220.
• Most serious relative contraindication for performing a pulmonary angio:
• Pulm arterial pressure 25/12
• L BBB
• R BBB
• Swan-Ganz catheter
• IVC filter
• B. LBBB.The relative contraindication for performing pulmonary angiogram is listed below:
• Pulmonary Hypertension:
defined as Systolic pulmonary artery pressure > 30mmHg or mean PA pressure > 18 mmHg LBBB
Catheter manipulation in RA  may produce RBBB  which then leads to total heart block
If the clinician “really wants the study,” a transvenous pacer might be placed.
• Allergic contrast reaction
• Premedicate with steroids
• Primer, 3rd, 688-689 and Kandarpa, Handbook of IR, pp 42-43
• Drainage of an abdominal abscess.
• a. The drainage tube must be under suction.
• b. The tube must be removed immediately after drainage has stopped.
• c. Can cause sepsis and DIC with manipulation of the catheter
• C. can cause sepsis and DIC wioth manipulation of the catheter. Overall complications of abscess drainage is less than 10%. The main complications are sepsis, tube migration/dislodgment, bowel perforation, hemorrhage, and peritonitis. To minimize complications avoid transversing loops of bowel, minimize wire/catheter manipulation, and irrigate cavities gently to minimize septicemia. Synopsis of Castaneda, 391
• Reason for not performing a TIPS procedure (T/F):
• Intractable ascites
• Failed sclerotherapy
• Isolated splenic vein thrombosis
• Pleural effusion
• a) False Although intractable ascites is not considered a standard indication for a TIPS procedure, there are a few centers which are currently evaluating its efficacy in this subgroup of patients. Therefore, this would be considered an unproven but promising use for TIPS.b) False Failed sclerotherapy is the primary indication for performing a TIPS. Other medical interventions commonly used in the treatment of variceal hemorrhage include vasoconstricting drugs and the use of gastroesophageal balloon placement. c) True In a patient with isolated splenic vein thrombosis without concomitant portal hypertension, there is no benefit in performing a TIPS procedure. Splenic vein occlusion is most commonly secondary to pancreatitis, pancreatic pseudocyst, or pancreatic carcinoma, but can be caused by retroperitoneal lesions or hyperviscosity states such as polycythemia.d) True An isolated pleural effusion is not an indication for a TIPS. However, if the pleural effusion is a secondary manifestation of severe hepatic dysfunction with associated portal hypertension and volume overload, then a TIPS may be considered. ABSOLUTE CONTRAINDICATION FOR TIPS
• Right-sided heart failure with elevated central venous pressuresPolycystic liver diseaseSevere hepatic failureLarge hepatocellular carcinoma infiltrating the porta hepatis, precluding stent placementRelative Contraindication For Tips
• Active intrahepatic or systemic infectionSevere hepatic encephalopathy poorly controlled by medical therapy Portal vein thrombosisPrimer, 3rd, 653-654
• PTA is not first procedure of choice in:
• Fresh clot
• 5 cm long stenosis of the SFA
• tight stenosis of the iliac artery
• A Fresh clot. (best answer)In a patient with a fresh clot (a), PTA is an absolute contraindication because of a risk of downstream embolization unless thrombolytic agents are used before hand to lyse the fresh clots. Lesions longer than 4.0 cm in the SFA (b) is also considered a relative contraindication (p379). A tight stenosis of the iliac artery (c) is considered amendable to PTA (see below).Relative Contraindications to PTA:
Aorta- eccentric lesions in prox. Infrarenal region
Iliac- stenoses longer than 2.0-3.0 cm, eccentric stenoses, heavy calcifications, total occlusion of common ilica artery, or stenosis at the origin of the common iliac artery.
Superficial and deep femoral artery- stenoses longer than 2.0-3.0 cm, occlusions longer than 10-15 cm, or heavy calcifications of the stenotic or occluded region.
Popliteal, tibial and peroneal arteries-long segments of disease where there is a large patent vessel at the ankle, because the latter vessel is suitable for bypass grafting, good results are likely to be obtained, particularly in institutions where in situ grafting is utilized.Others:
-Ulceratice disease with evidence of distal emobolization.
-Complete occlusion of SFA in presence of morbid obesity.
-Extensive disease in any segment of aortoiliac or femoral vasculature, because long segments of intimal-medial disruption predispose to recurrent disease and distal complications.
-Fresh mural thrombus unless thrombolysis is successful.
-Poor runoff diminishes the likelihood of successful PTA. Lesions ideal for angioplasty:
-focal distal aortic stenoses of atherosclerotic origin
-1.0 to 2.0 cm concentric, high grade stenoses of the iliac arteries located some distance from a major bifurcation.
-2.0 to 4.0 cm stenoses or occlusions of the SFA and DFA
-Relatively short isolated stenoses or occlusions in the popliteal and tibial arteries.Primer, 3rd, 651-652 and IR, Casteneda-Zuniga, 2nd, pp.379-380 (or 3rd edition, pp.502-504)
• Regarding percutaneous biliary drainage, which is true:
• a. parenteral antibiotics are required if pt is septic or not
• b. if you have an isolated obstructed right ductal system, the best entry site is in central right duct
• c. usually indicated for pancreatic carcinoma with obstructive jaundice
• A. parenteral antibiotics are required if pt. is septic or not.
• Prophylaxis Necessary for the following IR procedures: Biliary drainage, percutaneous nephrostomy, stent placement through a previously placed sheath or a groin hematoma, liver chemoembolization, splenic embolization, high risk for endocarditis.Prophylaxis recommended: TIPS, ports, pts with aortobifemoral bypass undergoing angiography through a femoral approach.Prophylaxis controversial: Procedure > 3 hours, placement of dialysis catheters, percutaneous gastrostomy tube placement, percutaneous drainage procedures.Prophylaxis unnecessary: diagnostic angiography, angioplasty, routine stent placement, TIPS revision, IVC filter placement. Synopsis of Castaneda’s 4-5
• Drugs used to treat the following:
• 1. Arterial vasospasm
• 2. Vasovagal reaction
• 3. Severe bronchospasm

• a. Epinephrine
• b. Lidocaine
• c. Nitroglycerine
• d. Atropine
• e. Narcan
• 1=CArterial spasm (angina): Nitroglycerin
• 2=DVasovagal Reaction: Isotonic saline, atropine
• 3=ABronchospasm: Beta-agonists, epinephrine, aminophylline
• Primer, 3rd, 950
• The following arteries branch off of what other artery?
• 1. Inferior pancreaticoduodenal artery
• 2. cystic artery
• 3. Right gastroepiploic

• a. Celiac
• b. SMA
• c. Splenic artery
• d. Right hepatic
• e. Gastroduodenal artery
• The following arteries branch off of what other artery?
• 1. Inferior pancreaticoduodenal artery
• 2. cystic artery
• 3. Right gastroepiploic

• a. Celiac
• b. SMA
• c. Splenic artery
• d. Right hepatic
• e. Gastroduodenal artery

• 1=B 2=D 3=E

• Right gastroepiploic comes off gastroduodenal
• Netter plates 282, 283
• T/F regarding stress induced thrombosis of the subclavian vein:
• Made worse with strenous exercise
• Not associated with central venous catheter
• Occurs most often in older women
• Usually seen in body builders
• 1=T 2=T 3=F 4=T Stress induced thrombosis is the same thing as idiopathic axillary and subclavian vein thrombosis. It also goes by: gouty plebitis, spontaneous subclavian vein thrombosis, effort thrombosis of subclavian vein, primary axillary-subclavian vein thrombosis, and Paget-Schroetter syndrome. It is a distinct entity from subclavian vein thrombosis secondary to central catheter, malignancy and other causes. 2/3 of patients are male <40 yo. Most have history of vigorous exercise prior to onset of symptoms such as weight lifters. Presentations include cyanosis of the extremity, swelling, and prominent venous collateral about the shoulder and anterior chest. There are two venographic patterns: long segment obstruction of the axillary and subclavian veins; and short segment obstruction at the level of the 1st rib0clavicle junction anteriorlyCastenda-Zuniga, 3rd, p. 793-794
• If an occulsion occurs at the following level, what is the collateral pathway (Matching):
• 1. IVC inferior to the renal veins
• 2. Splenic vein
• 3. SVC just below the azygous

• a. Left gastric to portal
• b. Internal Iliac to ascending lumbar to azygous
• c. Azygous to ascending lumbar to IVC
• If an occulsion occurs at the following level, what is the collateral pathway (Matching):
• 1. IVC inferior to the renal veins
• 2. Splenic vein
• 3. SVC just below the azygous

• a. Left gastric to portal
• b. Internal Iliac to ascending lumbar to azygous
• c. Azygous to ascending lumbar to IVC

• 1=b 2=a 3=cYou just have to look at the anatomy atlas for there is no better explanation.Netter, plates 226, 248, 297
• Embolization of pulmonary AVMs in a child should be performed with:
a. Boiling contrast
b. Ivalon particles
c. Coils
d. Alcohol
e. Gelfoam
• Coils.
• Which is false concerning peripheral AVF?
• a. Many spontaneously close
• b. Increased pulsatility in the vein
• c. Increased velocity in the artery distal to the fistula
• d. Low resistance waveform in the artery feeding the AVF
• C. Increased velocity in the artery distal to the fistula.
• Severe chest pain is experienced during a dipyridamole study. One should:
• stop infusion of dipyridamole and give nitroglycerine
• stop infusion of dipyridamole and give aminophylline
• stop the infusion of dipyridamole and wait
• continue with the exam
• decrease the infusion rate and titrate until the pain subsides
• B. stop infusion of dipyridamole and give aminophylline.
• Aminophylline will reverse the effects of dipyamidole.Primer 897
• Concerning abdominal aortic aneurysyms (multiple true/false).
• Aneurysyms larger than 7.5 cm are associated with a 95% chance of rupture within 5 years
• They are more common in black males than in white males
• They are more common in females than in males
• The size of the aneurysm is better assessed with angiography than with contrast-enhanced CT
• Bilateral renal arterial involvement precludes surgery
• Concerning abdominal aortic aneurysyms (multiple true/false).
• Aneurysyms larger than 7.5 cm are associated with a 95% chance of rupture within 5 years
• They are more common in black males than in white males
• They are more common in females than in males
• The size of the aneurysm is better assessed with angiography than with contrast-enhanced CT
• Bilateral renal arterial involvement precludes surgery


• False Complications of AAA include, Rupture, Peripheral embolization, Infection, Spontaneous occlusion of Aorta. The estimated risk for rupture in 5 years for a 7.5 cm AAA is ~70% and about 50% of 7cm AAA’s will rupture in 5 yrs
• False RISKS: White male >> Black male ~ Black female ~ White female M:F = 5:1
• False
• False AAA = aortic lumen greater then 3 cm or twice the lumen of the normal aorta. CT: -Most accurate in determining size of aneurysm. Also the best study to detect suspected rupture. Angiography: -Poor for size determination due to mural thrombus. AAA size can be undersized. Yields more accurate information regarding status of mesenteric & renal vessels.
• False Bilateral renal involvement would not preclude surgery but would change the surgical approach. It would become more involved and require implantation of visceral vessels including the renal arteries to the prosthetic graft. It would however preclude endoluminal repair with a stent graft.Reference: Vascular Surgery, vol. II of II, 1994. Robert Rutherford. Dähnert 605-6Primer 670
• Concerning PTA (multiple true/false):
• Stenting is indicated for iliac dissection caused by PTA
• Balloon size is determined by degree of stenosis
• For recurrent stenosis of popliteal and posterior tibial arteries, stenting of these vessels is indicated
• Injury to the intima is associated with a poor prognosis
• Pain lasting greater than one hour following PTA is associated with a poor prognosis
• True Indicated for ineffective or unsuccessful PTA such as large post dissection flap
• False Balloon sized to adjacent normal artery
• False, small vessel size
• False Angioplasty is controlled intimal injury.
• True. Continued post procedure pain may herald vascular rupture. Brant & Helms 616-619 Primer 651-653
• Concerning vasculidities (matching):
• 1) Plethora and upper extremity head & neck swelling
• 2) 29-year-old female with prominent arc of Riolan and buttock claudication
• 3) Unilateral upper extremity hypertension
• a. Buerger's disease
• b. Takayasu's disease
• c. Behcet's disease
• d. Kawasaki's disease
• e. Scleroderma
• 1. C. Behcet's Disease = a rare multisystem disease of unknown origin characterized by apthous stomatitis, genital ulceration, and iritis. Positive pathergy test = unusual hypersensitivity to pricking with formation of pustules at the site of needle prick within 24-48 hours. May cause SVC syndrome.
• 2. B. Takayasu's Arteritis = Pulseless Disease = Aortitis Syndrome = chronic inflammatory panarteritis of unknown pathogenesis affecting segments of the aorta, main aortic branches and pulmonary arteries. This is the only form of aortitis that produces stenosis/occlusion of the aorta. Typical scenario is a young female with decreased or absent pulses, blood pressure discrepancies, arterial bruits. Acute stage: granulomatous infiltration of elastic fibers of media Fibrotic stage (weeks to years): progressive fibrosis of vessel wall resulting in constriction from intimal proliferation/thrombotic occlusion/aneurysm formation. Ultimately leads to fibrosis of intima & adventitia.
* an Arc of Riolan (inconstant anastomosis) runs between the superior and inferior mesenteric arteries, and would provide collateral flow in case of aortic stenosis. Named for Jean Riolan, 1580-1657, French physician.
• 3. A. Buerger's disease = Thromboangiitis Obliterans = idiopathic, recurrent, segmental obliterative vasculitis of small and medium sized peripheral arteries and veins (panangiitis). Incidence is <1%, more common in Israel, Orient, and India.. Acute stage: see multiple microabscesses within fresh/organizing thrombus. PATHOGNOMONIC: multinucleated giant cells within microabscesses. Subacute stage: thrombus formation with little residual inflammation Chronic stage: lumen filled with organized, recanalized thrombus, fibrosis of adventitia binds together artery, vein and nerve. 95% association with cigarette smoking. Angiography reveals arterial occlusions, tapered narrowing of arteries, abundant corkscrew collaterals, Martorell sign (direct collateral following the path of the original artery (80%)), skip lesions, and absence of generalized athersclerosis.
• Kawasaki affects children. Coronary artery abnormality. Dähnert 461, 616, 634, 648
• Dissection of the aorta is least likely to have which of the following vessels arising from the false lumen (single best answer):
• Left renal artery
• Left subclavian artery
• Left iliac
• Right carotid
• Left coronary
• E. Left coronary.
• Indications for the TIPS procedure include (multiple true/false):
• Failed sclerotherapy with esophageal varices
• To improve hepatic encephalopathy
• Used for treatment of gastric varices
• Used for unmanageable ascites
• Indicated in the setting of portal hypertension
• True. Kandarpa gives failed endoscopic correction of variceal bleeding in patients with portal hypertension as the primary indication.
• False. TIPS may well worsen hepatic encephalopathy.
• True. If the varices are a source of intractable bleeding.
• True. Intractable ascites listed as potentially benefiting.
• True. Potential benefit of decompressing portal circulation prior to surgery
• Dähnert 735-736
• Prostaglandin therapy for treatment of PDA is associated with which of the following:
• periostitis
• osteomalacia
• osteopenia
• soft tissue swelling
• fracture
• A. periostitis
• Dähnert 12
• Thirty-five-year-old woman suffers acute onset of unilateral upper extremity hypertension and signs of stroke/CNS ischemia. Most likely etiology includes:
• Takayasu arteritis
• Giant cell arteritis
• Buerger disease
• Kawasaki disease
• A Takayasu’s arteritis. See previous.
• Regarding GI bleeding studies:
• Need at least 1.5 ml/min blood loss for a tagged RBC study to detect a bleed
• Tagged RBC study is about as sensitive as a sulfur colloid study
• Twenty-four hour image in a tagged RBC study is useful to see the appearance of movement of tracer
• To identify a GI bleed, you must see tracer in the lumen and movement of the tracer
• A. False.
• B. False. GI bleeding scans need a rate of minimum 0.1 mL/min for detection (0.05 mL/min according to Dähnert) (compared to a minimum of 0.5 mL/min for angiography). Tagged red cell studies are more sensitive than sulfur colloid studies. Sulfur colloid has several disadvantages: Bleeding must be active at time of administration (with tagged cell study, bleeding may or may not be active, and allows detection over a long period of time), it is cleared very rapidly form blood by the RES (not allowing for imaging over a long period, although that is an advantage in the unstable patient in which rapid diagnosis is needed, since the whole study only takes 20 min versus 1h for tagged cell study), and it is not useful for upper GI bleeding (interference from high activity from liven and spleen) or bleeding near hepatic/splenic flexures of colon.
• C. True. Twenty four hour image in a tagged red cell study may be useful to see movement of tracer/change in appearance of tracer, consistent with bowel peristalsis. In fact, up to 33% of studies may become positive only after 12‑24 h. Single best answer would be:
• D. True. To identify a GI bleed (the key word here is ACTIVE BLEEDING, which was not put in the distracter) the tracer must be seen in the bowel and move antegrade and/or retrograde with peristalsis. It is essential for diagnosis that the activity moves within bowel. Fixed activity is unlikely to represent active bleeding. (tracer accumulates with time, conforms to bowel anatomy and then moves). Tagged red cell studies are longer to prepare and perform, and have the potential for false positive resultsReference: Dähnert 1111-1112
• The left gastroepiploic artery is a branch of the:
• Splenic artery
• Gastroduodenal artery
• Superior mesenteric artery
• Right gastric artery
• Left gastric artery
• Celiac artery
• A. left comes off splenic, right comes off gastroduodenal
• The left gastroepiploic artery is a branch of the splenic artery. Primer 675
• True/False - Regarding aortic dissection:
• MRI is the imaging study of choice
• The false lumen does not fill with contrast on CT
• Answers:
• True. Advantages of MRI: Allows imaging of entire aorta from the aortic valve to the aortic bifurcation (precise knowledge of extension of dissection) which is useful for surgical planning and for follow‑up of stability Versus progression after either surgical or medical treatment; detects aortic valve regurgitation; allows evaluation of blood flow in the true and false channels; multiplanar capabilities. Comparing MRI, CT and TEE (transesophageal echo) for ascending aorta: sensitivities of 100%, 82.6% and 96.4%; and specificities of 98.6%, 100% and 85.7% respectively.
• False. The false lumen can fill with contrast on a CT study, and this may be simultaneous with flow in the true lumen or delayed, depending on flow characteristics
• Dähnert 607-609
• True/False - Regarding FMD of the carotid artery:
• The internal carotid is affected more frequently than the external carotid
• The false lumen does not fill on angiography
• You see a string of beads sign on angiography
• The intracranial vessels are also affected
• It is usually unilateral
• True. The internal carotid is involved more frequently than the external carotid, in fact, it is the most frequently affected vessel in the neck. (The order of involvement counting all arteries of the body is: renal artery by far the most common, internal carotid, external iliac). Involvement begins approximately 2 cm distal to the bulb in most cases, and rarely affects the bulb.
• False. According to Osborne, the false lumen may rarely fill. In the overwhelming majority of cases, the dissection is seen angiographically as a narrowing of the true lumen, with no visualization of the false lumen nor of an intimal flap of dissection.
• True. The string of beads is the most common angiographic sign of FMD, in cases of FMD of the media, which is the most common variety. The other Two varieties can also be seen (fibroplasia of the intima or of the adventitia, in which cases, isolated short narrowings or long smooth narrowings can be noted angiographically).
• True. Affected less frequently than extracranial vessels.
• False. FMD is bilateral in 60‑75%.References: 1) Diagnostic Neuroradiology. Ann Osborne 1994; 2) Surgical Neuroangiography by Lasjaunias and Berenstein 1992. Dähnert 629
• True/False - Regarding arterial stenting:
• It is the treatment of choice for dissection after PTCA
• The size of the balloon is indicated by the severity of stenosis
• It is indicated for popliteal and perineal artery stenosis
• True. Stenting is the treatment of choice for dissection after PTCA. This is the single best answer. Conditions for stenting include a significant pressure gradient or flow‑limiting dissection (flow is asymmetric, slower in relation to the uninvolved site). Not all dissections need to be stented, particularly small or nonflow limiting ones.
• False. The size of the balloon is determined by the diameter of the adjacent noninvolved vessel, without correction for magnification on the films
• False. Surgery has better long term patency results than percutaneous interventions for infrainguinal vessel disease, due mainly to the smaller size of these vessels. Stents can be done in these vessels, Mostly when patients are not good surgical candidates.Reference: Interventional Radiology, 3rd Edition 1997. Castañeda‑Zúñiga. Brant & Helms 616-619. Primer 651-653
• True/False - Which of the following veins have the valves?
• Azygous vein
• Coronary vein
• Cephalic vein
• Spermatic vein
• True. "A few imperfect valves are found in the azygous vein, but its tributaries are provided with complete valves.
• "True (assuming this means coronary sinus). "It (the coronary sinus) ends in the right atrium between the opening of the IVC and the right atrioventricular orifice, its opening displaying a semilunar flap, the valve of the coronary sinus.
• "True. "Both sets (the deep and superficial veins of the upper extremity) are provided with valves, which are more numerous in the deep than in the superficial system."(No formally named vein, but likely refers to the pampiniform plexus and testicular veins.)
• False. "The testicular vein and the pampiniform plexus are devoid of valves except for an occasional one at their termination in the IVC or the left renal vein."Reference: Textbook of Anatomy, 4th Edition, Hollinshead and Rosse. Reference: Grays Anatomy, 36th Edition.
• True/False - In a trauma patient, displacement of the nasogastric tube on a chest x-ray is sensitive or specific for aortic transection.
• Answer: Sensitivity: 12-100%; Specificity:80-95%.NG Tube deviated to the right at T3-T4 more specific for aortic injury.Reference: Dähnert 617